File:  [Local Repository] / db / baza / sibst16.txt
Revision 1.7: download - view: text, annotated - select for diffs - revision graph
Sat Nov 10 13:01:42 2018 UTC (5 years, 6 months ago) by rubashkin
Branches: MAIN
CVS tags: HEAD
*** empty log message ***

Чемпионат:
Студенческая лига Сибири - 2016/17

Дата:
30-Sep-2016 - 24-Mar-2017

Тур:
1 тур

Дата:
30-Sep-2016

Редактор:
Тарас Вахрив (Тернополь)

Инфо:
Редактор благодарит за тестирование и помощь в подготовке пакета Дмитрия
Алёхина, Евгения Быстрова, Ирину Вахрив, Дмитрия Великова, Садига
Гамидова, Андрея Данченко, Ирину Зубкову, Артема Корсуна, Александра
Кудрявцева, Сергея Лобачёва, Юлию Лободу, Юлию Лунёву, Дмитрия Овчарука,
Александра Островского, Андрея Остроглазова, Юрия Рудыка, Андрея Цяпу,
Евгения и Марию Шляховых.

Вопрос 1:
Согласно китайской легенде, Солнце и Луна занимаются ЭТИМ. Некоторых
участников ЭТОГО называют "якорями". Назовите ЭТО.

Ответ:
Перетягивание каната.

Комментарий:
Солнце и Луна с переменным успехом перетягивают канат, из-за чего
происходит смена дня и ночи. Последних участников команд называют
якорями, поскольку они стопорят движение вперед.

Источник:
   1. http://en.wikipedia.org/wiki/Tug_of_war
   2. http://www.sport-express.ru/newspaper/2005-07-18/14_2/

Автор:
Тарас Вахрив (Тернополь)

Вопрос 2:
На логотипе службы доставки ОНИ изображены в виде двух кусочков пиццы.
Иногда при ИХ описании упоминается X [икс]. Назовите ИХ двумя словами.

Ответ:
Песочные часы.

Комментарий:
Песочные часы на логотипе символизируют быстроту доставки. Буква X [икс]
визуально напоминает песочные часы.

Источник:
   1. https://logobaker.ru/logo/5636-pizza-time.html
   2. http://www.corsetplus.ru/figura-pesochnye-chasy/

Автор:
Тарас Вахрив (Тернополь)

Вопрос 3:
   <раздатка>
   Appletters
   </раздатка>
   Американец Эйб НАтансон придумал лингвистическую игру, в которой
нужно анаграммировать слова из наборов букв. Игра стала популярной и
обзавелась разновидностью, название которой приведено на раздатке. В
форме какого плода сделан чехол для костяшек с буквами в изначальной
версии игры?

Ответ:
Банан.

Комментарий:
Игра называется "Bananagrams" [бананагрАмс], и ее целью является
анаграммирование слов. Названия "Bananagrams" [бананагрАмс] и
"Appletters" [эплЭттэрс] являются словами-бумажниками.

Источник:
http://en.wikipedia.org/wiki/Bananagrams

Автор:
Тарас Вахрив (Тернополь)

Вопрос 4:
Компания "TEPCO" [тЭпко] некоторое время назад понесла огромные потери.
Выплата компенсаций более чем 80 тысячам переселенцев и снижение
стоимости акций на 80% поставило компанию на грань распада. Назовите
страну, в которой базируется упомянутая компания.

Ответ:
Япония.

Комментарий:
Финансовые дела разладились после аварии на принадлежащей компании АЭС
Фукусима I [один]. Слово "распада" в тексте вопроса - подсказка.
Название компании - акрОним от "Tokyo Electric Power Company" [тОкио
элЕктрик пАуэр кАмпани].

Источник:
   1. http://ru.wikipedia.org/wiki/Tokyo_Electric_Power_Company
   2. http://ru.wikipedia.org/wiki/Авария_на_АЭС_Фукусима-1

Автор:
Тарас Вахрив (Тернополь)

Вопрос 5:
В детском произведении лев использует ЕГО в качестве трона. Некий БАртон
вызвал возмущение природозащитной организации, поскольку в качестве
основания для беседки использовал ЕГО. Назовите ЕГО двумя словами,
которые начинаются на парные согласные.

Ответ:
Пень баобаба.

Комментарий:
Пни баобабов огромные, поэтому царский размер хорошо подошел для трона
льву.

Источник:
   1. http://ped-kopilka.com.ua/vneklasnaja-rabota/zagadki-schitalki-i-skorogovorki/zagadki-dlja-detei-7-8-let-s-otvetami.html
   2. http://www.ardiss.ru/o-nas/poleznaya-informacziya/stati-po-teme-stroitelstva/ustroystvo-fundamenta-s-setkoy-dorozhnoy-i-bez-nee/

Автор:
Тарас Вахрив (Тернополь)

Вопрос 6:
Ученые предполагают существование еще одной аллотропной формы
химического элемента, которая получила название ИКС по аналогии с
некоторыми другими модификациями. На натюрморте Михаила Врубеля часть
ИКСА закрыта стаканом. Что мы заменили на ИКС?

Ответ:
Графин.

Комментарий:
Аллотропная форма углерода получила название по аналогии с уже
существующими графитом и графеном. Кстати, этот вопрос был шестым, как и
номер углерода в периодической системе.

Источник:
   1. http://en.wikipedia.org/wiki/Graphyne
   2. http://ru.wikipedia.org/wiki/Аллотропия_углерода
   3. http://www.art-catalog.ru/picture.php?id_picture=1410

Автор:
Тарас Вахрив (Тернополь)

Вопрос 7:
В 2015 году, когда уже утихли страсти, латиноамериканские пивовары
выпустили довольно крепкое пиво, в названии которого упоминалась
европейская страна. Назовите с точностью до десятых процента содержание
алкоголя в упомянутом пиве.

Ответ:
7,1%.

Зачет:
7,1.

Комментарий:
Пиво "Гол Германии" служит напоминанием о провальном матче сборной
Бразилии против Германии на чемпионате мира по футболу 2014 года,
который закончился разгромом латиноамериканцев со счетом 7:1.

Источник:
http://www.umgoleoumais.com/cerveja-gol-da-alemanha-possui-71-de-teor-alcoolico/

Автор:
Тарас Вахрив (Тернополь)

Вопрос 8:
ЭТА английская фамилия происходит от названия профессии человека,
который в Средние века следил за заповедниками феодала. Что производит
компания, основанная носителем ЭТОЙ фамилии в 1888 году?

Ответ:
Ручки.

Зачет:
Авторучки; пишущие инструменты; чернильные ручки.

Комментарий:
Фамилия - Паркер. В Средние века парками назывались заповедники
феодалов, в которых охотились на оленей.

Источник:
   1. http://news.flarus.ru/?topic=6665
   2. http://en.wikipedia.org/wiki/Parker_Pen_Company

Автор:
Тарас Вахрив (Тернополь)

Вопрос 9:
На рекламном билборде IKEA [икЕа] изображен угловой диван, который
хорошо вписывается в меблирОванную комнату. Какое детище уроженца Москвы
вдохновило дизайнеров на создание описанного плаката?

Ответ:
Тетрис.

Комментарий:
(pic: 20160739.jpg)
   Билборд символизирует стакан "Тетриса", заполненный фигурами. Диван -
новая L-образная фигура, которую можно поставить в единственное
подходящее место.

Источник:
   1. https://twitter.com/brilliant_ads/status/840942224347189248/
   2. http://en.wikipedia.org/wiki/Tetris

Автор:
Тарас Вахрив (Тернополь)

Вопрос 10:
В известной серии комиксов для восстановления потерянной во время боя
памяти к НЕЙ подключили жесткий диск. В другой серии произведений ЕЕ
отделил Лед. Назовите ЕЕ двумя словами.

Ответ:
Голова Старка.

Комментарий:
В комиксах о Железном Человеке мозг Тони Старка подключался к
компьютеру. В "Песне льда и пламени" Илин Пэйн казнил Нэда Старка
фамильным мечом последнего, который назывался Лед.

Источник:
   1. http://ru.wikipedia.org/wiki/Железный_человек
   2. http://ru.gameofthrones.wikia.com/wiki/Казнь_Эддарда_Старка

Автор:
Тарас Вахрив (Тернополь)

Вопрос 11:
[Ведущему: четко прочитать слово "цИлий", чтобы игроки не восприняли его
как "целей".]
   Поскольку волнообразные движения цИлий толкают ЕЕ тупым концом
вперед, у наблюдателя может сложиться впечатление, что ОНА движется
задом наперед. Назовите ЕЕ.

Ответ:
Инфузория-туфелька.

Зачет:
Инфузория.

Комментарий:
ЦИлии - другое название ресничек, с помощью которых передвигаются
инфузории. Свое название инфузория-туфелька получила из-за сходства с
формой подошвы туфли. Инфузории-туфельки двигаются не острым "носком"
вперед, а "пяткой".

Источник:
   1. http://ru.wikipedia.org/wiki/Реснички
   2. http://ru.wikipedia.org/wiki/Инфузория-туфелька

Автор:
Тарас Вахрив (Тернополь)

Вопрос 12:
[Ведущему: четко прочитать слово "Мыловаренной"].
   Центральным объектом Мыловаренной палаты Патриаршего дворца
Московского Кремля является огромная печь, сделанная в виде креста.
Какие две буквы мы заменили в предыдущем предложении?

Ответ:
ир.

Комментарий:
Речь в вопросе идет о Мироваренной палате.

Источник:
http://ru.wikipedia.org/wiki/Мироваренная_палата

Автор:
Тарас Вахрив (Тернополь)

Вопрос 13:
Иногда для поощрения детей на НЕГО наносят надпись. В современной
российской сказке после использования апельсина ОНО засверкало, но
изображение не появилось. Назовите ЕГО двумя словами.

Ответ:
Дно тарелки.

Зачет:
Дно блюдечка; дно блюдца; дно блюда; дно миски.

Комментарий:
Чтобы дети активнее кушали, на дно некоторых тарелок наносят
поощрительные надписи. В сказке был неурожай яблок, а при использовании
апельсина блюдечко ничего не показало.

Источник:
   1. http://www.sostav.ru/news/2010/07/27/zar3/
   2. О. Новицкая, В. Новицкий. Требуется сообщник.
http://flibusta.is/b/248289/read

Автор:
Тарас Вахрив (Тернополь)

Вопрос 14:
На гербе западноевропейской страны можно насчитать четырнадцать ИХ.
Жан-КристОф ГранжЕ сравнил с НИМИ отреагировавшие на столкновение
корабельные тросы. Назовите ИХ двумя словами.

Ответ:
Струны арфы.

Комментарий:
Арфа - символ Ирландии. Тросы от столкновения зазвучали, словно струны
арфы.

Источник:
   1. http://en.wikipedia.org/wiki/Coat_of_arms_of_Ireland
   2. Ж.-К. Гранже. Пассажир.
https://books.google.ru/books?id=ocp7AAAAQBAJ&pg=PT546#v=onepage&q&f=false

Автор:
Тарас Вахрив (Тернополь)

Вопрос 15:
(pic: 20160740.jpg)
   В финальных титрах фильма "Первый мститель: Противостояние" имена
актеров отбрасывают необычные тени. Например, тень имени Криса Эванса,
сыгравшего Капитана Америку, изображена в виде щита со звездой. Что
символизирует приведенная на раздатке тень, которую отбрасывает имя
актера, сыгравшего одного из супергероев?

Ответ:
Муравьиную кислоту.

Зачет:
Метановую кислоту; формулу муравьиной кислоты; формулу метановой
кислоты; молекулу муравьиной кислоты; молекулу метановой кислоты.

Комментарий:
Тень отбрасывает имя Пола Радда, который сыграл Человека-муравья. Тень
представлена в виде символического изображения муравьиной кислоты,
формула которой - HCOOH [аш-це-о-о-аш].

Источник:
   1. Х/ф "Первый мститель: Противостояние" (2016), реж. Энтони Руссо,
Джо Руссо.
   2. http://ru.wikipedia.org/wiki/Муравьиная_кислота

Автор:
Тарас Вахрив (Тернополь)

Вопрос 16:
Форма ЭТОГО негативно повлияла на акустику, поэтому внутри были
дополнительно установлены специальные отражатели звука. ЭТО вдохновило
дизайнеров компании "Klim" [клим] на создание стильного мотоциклетного
шлема. Назовите ЭТО.

Ответ:
Сиднейский оперный театр.

Зачет:
Сиднейская опера; крыша сиднейского оперного театра; крыша Сиднейской
оперы.

Комментарий:
Необычная форма крыши ухудшила звучание, поэтому на потолках были
оборудованы отражатели.

Источник:
   1. http://ru.wikipedia.org/wiki/Сиднейский_оперный_театр
   2. http://www.advrider.com/index.php?threads/klim-f4-helmet-2-australian-thumbs-up.423448/

Автор:
Тарас Вахрив (Тернополь)

Вопрос 17:
После провального матча на Евро-2016 против сборной Уэльса в
департаменте Верхняя Гаронна журналисты "Спорт-Экспресса" назвали ЭТИМ
неологизмом футболистов сборной России. Напишите ЭТОТ неологизм.

Ответ:
Тулузеры.

Зачет:
Тулузер.

Комментарий:
Матч проходил в Тулузе - административном центре Верхней Гаронны.

Источник:
   1. http://www.sport-express.ru/newspaper/2016-06-21/1_1/
   2. http://ru.wikipedia.org/wiki/Верхняя_Гаронна

Автор:
Тарас Вахрив (Тернополь)

Вопрос 18:
Питер Брейгель Старший на одной из картин буквально воспроизвел
евангельскую сентенцию "Где сокровище ваше, там ЭТО". Поэтому туго
набитый кошелек неблагочестивого монаха изображен в виде ЭТОГО. Назовите
персонажа другого произведения, который получил ЭТО.

Ответ:
Железный Дровосек.

Зачет:
Жестяной Дровосек.

Комментарий:
ЭТО - сердце. Согласно Библии, если сердце человека расположено к
богатству, то ему не попасть в Царство Небесное.

Источник:
   1. http://valya-15.livejournal.com/96292.html
   2. http://blagovest.al.lg.ua/9802/02.html
   3. http://ru.wikipedia.org/wiki/Железный_Дровосек
   4. http://en.wikipedia.org/wiki/Tin_Woodman

Автор:
Тарас Вахрив (Тернополь)

Вопрос 19:
Один из НИХ получил название "Сталактиты", поскольку постепенное
формирование восьми столбцов напоминает рост сталактитов. Назовите ЕГО
словом французского происхождения.

Ответ:
Пасьянс.

Источник:
http://en.wikipedia.org/wiki/Stalactites_(solitaire)

Автор:
Тарас Вахрив (Тернополь)

Вопрос 20:
Недавно австралийские ботаники вывели новый сорт паслёновых, который
способен плодоносИть в сложных условиях пустынь. По фамилии персонажа
какого произведения был назван выведенный сорт?

Ответ:
"Марсианин".

Комментарий:
Выведенный сорт был назван "паслёновые Уотни". Ботаник Марк Уотни выжил
на Марсе благодаря тому, что сумел вырастить картофель в сложных
условиях.

Источник:
http://hi-news.ru/eto-interesno/botanik-iz-marsianina-vdoxnovil-avstralijskix-uchenyx.html

Автор:
Тарас Вахрив (Тернополь)

Вопрос 21:
В одной статье рассказывалось о чешуекрылых вредителях, которые весьма
изобретательны в способах причинения вреда. Напишите название этой
статьи, которое отличается от известной поговорки замененной первой
буквой.

Ответ:
"Моль на выдумки хитра".

Комментарий:
Речь о яблонной моли. Перефразирована поговорка "Голь на выдумки хитра".

Источник:
   1. http://newspaper.moe-online.ru/view/232288.html
   2. http://ru.wikipedia.org/wiki/Яблонная_моль
   3. http://slovarick.ru/277/

Автор:
Тарас Вахрив (Тернополь)

Вопрос 22:
Персонаж мультфильма "Зверополис" говорит разочарованной крольчихе,
чтобы она не ДЕЛАЛА ЭТОГО. В российской армии разрешается ДЕЛАТЬ ЭТО при
температуре воздуха -10°C [минус десять градусов Цельсия] и ниже. Что мы
заменили словами "ДЕЛАТЬ ЭТО"?

Ответ:
Опускать уши.

Комментарий:
В мультфильме - аллюзия на фразу "опускать руки". Шапка-ушанка - один из
элементов зимней формы солдат.

Источник:
   1. Мультфильм "Зверополис" (2016), реж. Байрон Ховард, Рич Мур,
Джаред Буш.
   2. http://ru.wikipedia.org/wiki/Ушанка

Автор:
Тарас Вахрив (Тернополь)

Вопрос 23:
Первые монеты, которые использовались в Малайзии, имели насечки, поэтому
название современной малайзийской валюты - ринггИта - в переводе значит
"ТАКОЙ". Политолог Дмитрий ОлехнОвич считает, что ТАКОЙ гимн не является
показателем патриотизма. Что мы заменили словом "ТАКОЙ"?

Ответ:
Зазубренный.

Зачет:
Вызубренный.

Источник:
   1. http://ru.wikipedia.org/wiki/Малайзийский_ринггит
   2. http://rus.tvnet.lv/novosti/obschjestvo/215780-politolog_zazubrjenniy_gimn_strani_nje_pokazatjel_urovnja_patriotizma

Автор:
Тарас Вахрив (Тернополь)

Вопрос 24:
Индийские мечи урУми могут достигать шести метров в длину. УрУми
пользовались популярностью у убийц, поскольку легко маскировались под
НИХ. Какие ОНИ были приняты отстающей от цивилизованного мира Либерией
только в 1972 году?

Ответ:
Часовые [пояса].

Комментарий:
УрУми - гибкий меч, который обматывался вокруг талии и маскировался под
пояс. В Либерии до 1972 года не использовали систему часовых поясов, и
время в стране отставало от гринвичского на 44 минуты 30 секунд.

Источник:
   1. http://xage.ru/samoe-neobyichnoe-oruzhie-v-istorii-chelovechestva/
   2. http://ru.wikipedia.org/wiki/Часовой_пояс

Автор:
Тарас Вахрив (Тернополь)

Вопрос 25:
[Ведущему: максимально четко прочитать слово "Кон"].
   Победа англичан в битве при КресИ в 1346 году над превосходящими
силами французов стала возможной благодаря использованию длинных луков и
новой тактики. Многие историки считают битву при КресИ ИМ рыцарства.
"Кон" - тоже ОНО. Назовите ЕГО.

Ответ:
Начало конца.

Комментарий:
После битвы при КресИ рыцари потеряли преимущество, которое им давали
доспехи. Сначала убойные стрелы длинных луков, а позже огнестрельное
оружие привели к тому, что рыцари перестали быть грозной силой в битвах.

Источник:
http://www.smirnova-tatjana.ru/int-faktiistorii-anglii/319-bitva-pri-kresi.html

Автор:
Тарас Вахрив (Тернополь)

Вопрос 26:
Однажды Василий услышал рассказ о том, как деспот ВалихАн-торЕ приказал
казнить европейского путешественника и положить голову на НЕЕ. Руферы,
незаконно побывавшие на другой НЕЙ, обнаружили арабские и французские
надписи и сделали красочный фотоотчет. Назовите ЕЕ двумя словами.

Ответ:
Вершина пирамиды.

Комментарий:
По одной из версий, под впечатлением от услышанного художник Василий
Верещагин создал картину "Апофеоз войны", на которой изображена пирамида
черепов. Руферы лазят по крышам и другим высотным сооружениям.
Упомянутые в вопросе руферы забрались на вершину пирамиды Хеопса.

Источник:
   1. http://ru.wikipedia.org/wiki/Верещагин,_Василий_Васильевич
   2. http://www.istpravda.ru/pictures/5602/

Автор:
Тарас Вахрив (Тернополь)

Вопрос 27:
(pic: 20160741.jpg)
   Американский дизайнер Джи Ли визуализирует слова с помощью букв.
Часть какого слова представлена на раздатке?

Ответ:
Selfie [чтецу: сЕлфи].

Зачет:
Селфи.

Комментарий:
Буква F [эф] символизирует человека, буква I [ай] - смартфон.

Источник:
https://www.instagram.com/p/zSXDqPDbwF/

Автор:
Тарас Вахрив (Тернополь)

Вопрос 28:
Слова "ГЕРОЙ" и "ВТОРОЙ" в вопросе являются заменами.
   В недавнем фильме ЭТОТ персонаж представляется словами: "Я - ГЕРОЙ,
мой цвет - ВТОРОЙ". Назовите ЭТОГО персонажа.

Ответ:
Микеланджело.

Зачет:
Майк; Майки.

Комментарий:
Микеланджело - один из главных героев фильма "Черепашки-ниндзя 2". В
одном из эпизодов он представляется словами: "Я - Микеланджело, мой цвет
- оранжевый". Оранжевый - второй цвет радуги, а также цвет повязок
Микеланджело.

Источник:
Х/ф "Черепашки-ниндзя 2" (2016), реж. Дэйв Грин.

Автор:
Тарас Вахрив (Тернополь)

Вопрос 29:
Автомобиль с усовершенствованной системой безопасности Audi R8 [Ауди эр
восемь] рекламировался под слоганом "ПРОПУСК 2,78 [две целых семьдесят
восемь сотых] секунды". Заполните пропуск пятью короткими словами.

Ответ:
От ста до нуля за.

Комментарий:
"От 100 до 0 за 2,78 секунды". У автомобиля инновационная тормозная
система. Обычно в рекламе автомобилей используется противоположный по
смыслу слоган, в котором указывается время разгона от нуля до ста.

Источник:
https://www.coloribus.com/adsarchive/prints/audi-100-to-0-21049655/

Автор:
Тарас Вахрив (Тернополь)

Вопрос 30:
Индейцы ваорАни называют ИХ по-разному - словами со значениями "обжора",
"соня" или "грязнуля". Какого персонажа романа 1859 года во многих
статьях называют ИМ?

Ответ:
[Илья] Обломов.

Комментарий:
ОНИ - ленивцы. Обломов - персонаж одноименного романа Ивана Гончарова.

Источник:
   1. http://en.wikipedia.org/wiki/Sloth
   2. http://www.google.ru/search?q=обломов+ленивец

Автор:
Тарас Вахрив (Тернополь)

Вопрос 31:
Состоящий из сюжетно разрозненных эпизодов фильм "Виктор Франкенштейн"
кинокритик сравнил с НИМ. ОНИ вдохновляли некоторых
художников-авангардистов. Назовите ИХ двумя словами.

Ответ:
Лоскутные одеяла.

Зачет:
Лоскутные полотна.

Комментарий:
Лоскутные одеяла похожи и на картины абстракционистов, и на чудовище,
созданное Франкенштейном.

Источник:
   1. http://igiss.net/2015/11/viktor-frankenshtejn-victor-frankenstein/
   2. http://ru.wikipedia.org/wiki/Лоскутное_шитьё

Автор:
Тарас Вахрив (Тернополь)

Вопрос 32:
БурАк ДанишмЕнд предлагает свои услуги не только обычным клиентам, но и
людям, страдающим болезнью Альцгеймера. Назовите профессию БурАка
ДанишмЕнда, который, судя по всему, не раз обращался к своим коллегам.

Ответ:
Татуировщик.

Зачет:
Мастер татуировок; тату-мастер; кольщик.

Комментарий:
Чтобы легче было найти потерявшегося в городе человека с болезнью
Альцгеймера или чтобы помочь ему вспомнить свое имя и адрес, мастер
предлагает делать больным соответствующие татуировки на видной части
тела. ДанишмЕнд имеет много татуировок, в том числе на спине, которые
вряд ли делал себе сам.

Источник:
   1. http://www.hurriyetdailynews.com/tattooist-offers-to-tattoo-names-of-alzheimer-patients-in-izmir.aspx?pageID=238&nID=71786
   2. https://www.facebook.com/burak.danismend

Автор:
Тарас Вахрив (Тернополь)

Вопрос 33:
В одном эпизоде популярного мультсериала показан бег с барьерами, во
время которого засыпают все зрители на трибунах. Назовите этот
мультсериал.

Ответ:
"Барашек Шон".

Комментарий:
В состязаниях принимают участие овцы. Прыгающие через препятствия овечки
даже без подсчитывания навевают сон на зрителей.

Источник:
http://www.youtube.com/watch?v=nlYzqw_jvLg

Автор:
Тарас Вахрив (Тернополь)

Вопрос 34:
Интересно, что еще в 1964 году МигЕль ФицджЕральд взял самолет и
отправился ТУДА. Установив на видном месте привезенный предмет, МигЕль
мирно улетел домой. Где побывал МигЕль ФицджЕральд?

Ответ:
Фолклендские острова.

Зачет:
Фолкленды; Мальвинские острова.

Комментарий:
Архипелаг является предметом территориального спора Аргентины и
Великобритании, что в 1982 году привело к военному конфликту. Аргентинец
с британской фамилией еще за 18 лет до Фолклендской войны выдвинул
требования вернуть территорию и слетал туда, чтобы установить флаг
Аргентины.

Источник:
http://ru.wikipedia.org/wiki/Фолклендский_инцидент_1966_года

Автор:
Тарас Вахрив (Тернополь)

Вопрос 35:
[Ведущему: сделать короткую интонационную паузу между частями слова
"БернкАстель-Кус", чтобы игроки не восприняли его как "БернкАстелькус"].
   В 1905 году к городу БернкАстель присоединили находящуюся на другом
берегу реки МозЕль деревню Кус. С чем в одном туристическом обзоре
сравнили мост через МозЕль, расположенный в нынешнем городе
БернкАстель-Кус?

Ответ:
Дефис [в названии города БернкАстель-Кус].

Зачет:
Тире.

Источник:
http://tochka-na-karte.ru/blog-items/Bundesblog/226-Bernkastel-Kus.html

Автор:
Тарас Вахрив (Тернополь)

Вопрос 36:
Статья Википедии сравнивает неаккуратно написанную на древнегреческой
вазе букву пси с НЕЮ. Назовите ЕЕ двумя словами.

Ответ:
Куриная лапа.

Зачет:
Лапа курицы.

Комментарий:
Визуально буква пси похожа на куриную лапу. Фразеологизм "как курица
лапой" означает неряшливый почерк.

Источник:
   1. http://ru.wikipedia.org/wiki/Пси_(греческий_алфавит)
   2. http://phrase_dictionary.academic.ru/1042/

Автор:
Тарас Вахрив (Тернополь)

Тур:
2 тур

Дата:
04-Nov-2016

Редактор:
Мишель Матвеев (Санкт-Петербург)

Инфо:
Редактор благодарит тестеров: Андрей Кокуленко, Борис Моносов, Анастасия
Мартынова, Яна Азриэль, Виталий Пронькин, Мария Беланкова, Александр
Кудрявцев, Илья Городецкий, Мария Ильина, Виктор Дзекановский, Александр
Зинченко, Садиг Гамидов, Александр Огнев, Игорь Мазин, Алексей Баев,
Юлия Лунёва, Владимир Городецкий, Никита Воробьёв, Ярослав Домб, Алексей
Акименко.

Вопрос 1:
Сын Михаэля Шумахера начал карьеру гонщика. В заметке об этом говорится,
что ему вряд ли удастся далеко продвинуться по пути отца, но, впрочем,
дорогу... Закончите эту мысль двумя словами.

Ответ:
Осилит едущий.

Комментарий:
Пословица "Дорогу осилит идущий" в данном случае была переиначена на
более подходящий для гонщика вариант.

Источник:
http://lenta.ru/photo/2015/04/11/budukakpapa/

Автор:
Мишель Матвеев (Санкт-Петербург)

Вопрос 2:
Герой Акунина проходит важное собеседование у недружелюбно настроенного
человека, которого почти полностью скрывает письменный стол. Назовите
имя того, с кем мысленно ассоциирует себя герой.

Ответ:
Руслан.

Комментарий:
Герой глядит на крупную голову оппонента и сравнивает себя с пушкинским
Русланом, сражающимся с Головой.

Источник:
Б. Акунин. Фантастика.
https://books.google.ru/books?id=5f8vAAAAQBAJ&pg=PT70#v=onepage&q&f=false

Автор:
Мишель Матвеев (Санкт-Петербург)

Вопрос 3:
Рассказывают, что причиной ЕЕ появления стала кружка Сталина с кофе.
Назовите ЕЕ максимально точно.

Ответ:
Кольцевая линия Московского метро.

Зачет:
По упоминанию Москвы и кольцевой линии.

Комментарий:
Согласно этой байке, Сталин поставил кружку с кофе на план будущего
московского метро, и она там оставила круглый коричневый след.

Источник:
http://www.anekdot.ru/id/784616/

Автор:
Мишель Матвеев (Санкт-Петербург)

Вопрос 4:
Один магазин аксессуаров назван фамилией, которую носили два известных
литературных родственника. Назовите их имена.

Ответ:
Бильбо, Фродо.

Зачет:
В любом порядке.

Комментарий:
Магазин называется "Бэггинс". Сумки - один из основных товаров, которые
там продаются, а остальные часто носят в сумке.

Источник:
http://www.baggins.ru

Автор:
Мишель Матвеев (Санкт-Петербург)

Вопрос 5:
Детям иногда сообщают фразу: "Артисту Биму циркуль дашь, его фамилия -
...". Какая же?

Ответ:
Жэ-Аш.

Зачет:
Же Аш; GH.

Комментарий:
Детям, занимающимся шахматами, предлагают по первым буквам слов этой
фразы запомнить последовательность латинских букв на шахматной доске.

Источник:
http://www.devir.ru/dev/info75.php

Автор:
Мишель Матвеев (Санкт-Петербург)

Вопрос 6:
Участвовавшим в Первой мировой войне кавказским горцам не понравилась
замена, которую сделали из уважения к их религиозным чувствам, и они
попросили вернуть джигита вместо курицы. Назовите этого джигита.

Ответ:
Святой Георгий.

Зачет:
Георгий Победоносец; Георгий.

Комментарий:
На государственных наградах для нехристиан изображения святых, в том
числе Георгия, были заменены на двуглавого орла. Но джигит кавказцам
нравился больше, чем сомнительная птица, а в религиозный смысл
изображения они, очевидно, не особо вникали.

Источник:
http://ru.wikipedia.org/wiki/Кавказская_туземная_конная_дивизия

Автор:
Мишель Матвеев (Санкт-Петербург)

Вопрос 7:
Один афоризм рекомендует остерегаться шофера, верящего в ЭТО. Назовите
растение, которое, как считали на Руси, обладает ЭТИМ.

Ответ:
Бессмертник.

Комментарий:
ЭТО - бессмертие.

Источник:
http://lib.ru/ANEKDOTY/KROTKIJ/nenapisannoe.txt

Автор:
Мишель Матвеев (Санкт-Петербург)

Вопрос 8:
Вячеслава Шварца тоже заинтересовал этот сюжет. На его картине мрачный
старик сидит в кресле, молодой человек лежит на постели, а несколько
монахов читают молитвы. Назовите этого старика.

Ответ:
Иван Грозный.

Зачет:
Иван IV; Иоанн Грозный; Иоанн IV.

Комментарий:
Как и знаменитая картина Репина, эта картина написана на сюжет убийства
Иваном Грозным его сына Ивана.

Источник:
https://commons.wikimedia.org/wiki/File:Wjatscheslaw_Grigorjewitsch_Schwarz_001.jpg

Автор:
Мишель Матвеев (Санкт-Петербург)

Вопрос 9:
В Интернете предложили заменить в известном произведении два слова на
"ток включённый". Напишите эти два слова.

Ответ:
Кот ученый.

Комментарий:
"И днем, и ночью ток включённый всё ходит по цепи кругом". Цепь - это
еще и набор элементов, по которым течет электрический ток.

Источник:
http://www.anekdot.ru/id/691184/

Автор:
Мишель Матвеев (Санкт-Петербург)

Вопрос 10:
Гипсикратия, наложница царя Митридата, демонстрировала такие мужество и
смелость, что царь называл ее... Как?

Ответ:
Гипсикрат.

Комментарий:
За эти доблестные качества царь приравнивал ее к мужчине.

Источник:
Плутарх "Сравнительные жизнеописания. Помпей".
http://ancientrome.ru/antlitr/t.htm?a=1439003200

Автор:
Мишель Матвеев (Санкт-Петербург)

Вопрос 11:
В Таджикистане используется слово "кайхоннавард", хотя своих
кайхоннавардов в Таджикистане нет и пока их появления не ожидается. А
какое слово с тем же значением используется в Китае?

Ответ:
Тайконавт.

Комментарий:
Слово означает человека, побывавшего в космосе. Слово "кайхоннавард" в
определенной степени созвучно словами "космонавт" и "тайконавт".

Источник:
http://ru.wikipedia.org/wiki/Космонавт

Автор:
Мишель Матвеев (Санкт-Петербург)

Вопрос 12:
В агломерации Табатинги и Летисии ОНА всегда открыта, но во время матча
чемпионата мира по футболу ЕЕ пришлось закрыть. Назовите ЕЕ двумя
словами, начинающимися на одну и ту же букву.

Ответ:
Государственная граница.

Комментарий:
Бразильский город Табатинга и колумбийский город Летисия - два города на
границе стран, образующие одну агломерацию. Бразильско-колумбийская
граница проходит прямо по улицам, и обычно ее можно пересекать без
всяких ограничений, но на время матча между сборными этих стран границу
закрыли во избежание инцидентов.

Источник:
http://tass.ru/sport/1294786

Автор:
Мишель Матвеев (Санкт-Петербург)

Вопрос 13:
Эксперт Джозеф Борковски предположил, что женщина просто потеряла
несколько зубов. Назовите имя, под которым известна эта женщина.

Ответ:
Джоконда.

Зачет:
Мона Лиза.

Комментарий:
По мнению Борковски, выражение лица Джоконды типично для человека,
потерявшего передние зубы.

Источник:
http://brightside.me/article/10-famous-paintings-with-hidden-secrets-74555/

Автор:
Мишель Матвеев (Санкт-Петербург)

Вопрос 14:
К герою произведения А. Картера приходят друзья, рисуют рожицы и пишут
добрые пожелания, например "Возвращайся скорей!". Где они это пишут?

Ответ:
На гипсе.

Комментарий:
Герой сломал ногу и вынужден находиться в больнице.

Источник:
К.И. Шапиро. Травматология и ортопедия в зеркале искусства. - СПб.,
1997.

Автор:
Мишель Матвеев (Санкт-Петербург)

Вопрос 15:
Порядковый номер ЖаннУ в списке величайших - всего лишь 32. Тем не
менее, именно с ЖаннУ до 2004 года связывали последнюю нерешенную
проблему ИХ. Назовите ИХ именем собственным.

Ответ:
Гималаи.

Комментарий:
ЖаннУ - 32-я по высоте вершина мира высотой 7710 метров. Несмотря на то
что это далеко не самая высокая гора, она очень трудна для восхождения,
а сложнейший маршрут на вершину ЖаннУ по центру северной стены называли
последней нерешенной проблемой Гималаев. В 2004 году она была покорена с
более чем двадцатой попытки. Кстати, это сделала российская экспедиция.

Источник:
   1. http://www.stadium.ru/news/16095
   2. http://www.alpklubspb.ru/zm/17.htm
   3. http://www.sport-express.ru/newspaper/2004-06-01/15_24/
   4. http://ru.wikipedia.org/wiki/Жанну

Автор:
Мишель Матвеев (Санкт-Петербург)

Вопрос 16:
Путешествуя по Японии, Жан Маре решил попробовать сакэ. После первой же
рюмки он ощутил эффект и спросил официанта, не слишком ли крепок
напиток. Тот ответил: "Не беспокойтесь, это обычное...". Что?

Ответ:
Землетрясение.

Комментарий:
У Маре всё закачалось перед глазами, но не из-за крепости сакэ. В
момент, когда Маре выпил рюмку, началось небольшое землетрясение, что
для Японии действительно является обычным делом.

Источник:
http://www.abhoc.com/arc_an/2003_03/188/

Автор:
Мишель Матвеев (Санкт-Петербург)

Вопрос 17:
В черновиках какого произведения сначала упоминаются Соколов, Воробьёв и
Петухов, а в конце - Вербицкий?

Ответ:
"Лошадиная фамилия".

Комментарий:
В черновом варианте рассказа Чехова герои вспоминали птичьи фамилии. В
итоге оказывалось, что нужная фамилия - Вербицкий, поскольку "птица
садится на вербу".

Источник:
http://ru.wikipedia.org/wiki/Лошадиная_фамилия

Автор:
Мишель Матвеев (Санкт-Петербург)

Вопрос 18:
Один сайт отметил, что даже ЕГО мускулистой шее тяжело будет выдержать
вес в шесть с лишним килограммов. Назовите ЕГО.

Ответ:
[Майкл] Фелпс.

Комментарий:
Если он одновременно наденет все свои 23 золотые олимпийские медали,
придется нелегко. Фелпс - рекордсмен по количеству золотых олимпийских
медалей и по количеству олимпийских медалей вообще.

Источник:
http://www.sports.ru/tribuna/blogs/russiateam/1018752.html

Автор:
Мишель Матвеев (Санкт-Петербург)

Вопрос 19:
В XIX веке инженер Линан де Бельфон, командовавший возведением плотин,
намеренно представил правителю неверные расчеты, из которых следовала
экономическая нецелесообразность использования материала ИХ. Назовите
ИХ.

Ответ:
Пирамиды.

Комментарий:
Правитель Египта Мухаммед Али предложил использовать камень из пирамид
для возведения плотин на Ниле. Чтобы спасти пирамиды, де Бельфон и
сделал эти расчеты. Де Бельфон был главным инженером при строительстве
Суэцкого канала, которое Мухаммед Али разрешил Фердинанду Лессепсу.

Источник:
http://ru.wikipedia.org/wiki/Линан_де_Бельфон,_Луи

Автор:
Мишель Матвеев (Санкт-Петербург)

Вопрос 20:
При строительстве Панамского канала сожгли лес и траву на территории 30
квадратных километров, осушили 80 гектаров болот и разбрызгали больше
полумиллиона литров масел. В результате ОНА перестала быть препятствием
для строительства. Назовите ЕЕ.

Ответ:
Малярия.

Зачет:
Желтая лихорадка.

Комментарий:
Эти меры были приняты для борьбы с малярийными комарами.

Источник:
http://ru.wikipedia.org/wiki/Панамский_канал

Автор:
Мишель Матвеев (Санкт-Петербург)

Вопрос 21:
Знакомая автора вопроса говорит о краске в случае новых и грязи в случае
старых. Этим она опровергает известное утверждение. Какое?

Ответ:
Деньги не пахнут.

Комментарий:
Так пахнут бумажные деньги. "Деньги не пахнут" - знаменитое
высказывание, приписываемое императору Веспасиану, не разбиравшему
способов пополнения казны. Впрочем, причина его действий была не в
личной жадности, а в трудной экономической ситуации, в которой
находилась империя, когда он пришел к власти.

Источник:
Наблюдение Александры Киричук.

Автор:
Мишель Матвеев, Александра Киричук (Санкт-Петербург)

Вопрос 22:
Эмиль Кроткий упоминает некую почтенную даму, не чуждую литературной
деятельности. В каком платье, согласно его каламбуру, она ходит?

Ответ:
В мемуаровом.

Комментарий:
Дама на склоне лет пишет мемуары, поэтому ходит не в муаровом, а в
мемуаровом платье.

Источник:
http://lib.ru/ANEKDOTY/KROTKIJ/nenapisannoe.txt

Автор:
Мишель Матвеев (Санкт-Петербург)

Вопрос 23:
Персонаж романа, действие которого происходит в России в начале XX века,
замечает, что страна пронизана сетью железных дорог, но лишь одна ПЕРВАЯ
на всю ВТОРУЮ снабжает Дальний Восток. Назовите ПЕРВУЮ и ВТОРУЮ
рифмующимися словами.

Ответ:
Артерия, империя.

Комментарий:
Персонаж сравнивает сеть железных дорог с кровеносной системой и
говорит, что на Восток, где идет война с Японией, ведет всего одна
артерия, и то чахлая и подверженная тромбам.

Источник:
Б. Акунин. Алмазная колесница. http://flibusta.is/b/175061/read

Автор:
Мишель Матвеев (Санкт-Петербург)

Вопрос 24:
В апреле 2007 года Сунита Уильямс, находясь на МКС, примерно 4,5 часа
использовала специальный тренажер. Таким образом она, как считается,
приняла участие в событии, проходившем... В каком американском городе?

Ответ:
Бостон.

Комментарий:
Уильямс как бы бежала на этом тренажере Бостонский марафон.

Источник:
http://www.vesti.ru/doc.html?id=96111

Автор:
Мишель Матвеев (Санкт-Петербург)

Вопрос 25:
Перед одним из матчей футбольного клуба "Леганес" чествовали его
102-летнего болельщика с самым большим фанатским стажем. Утверждается,
что он начал болеть за клуб в 14 лет, когда произошло некое событие.
Какое?

Ответ:
Основание "Леганеса".

Зачет:
Основание клуба; создание "Леганеса"; создание клуба.

Источник:
http://www.sports.ru/tribuna/blogs/saltis/1082701.html

Автор:
Мишель Матвеев (Санкт-Петербург)

Вопрос 26:
Описывая мертвого пирата, персонаж Стивенсона упоминает кое-что, смысла
в чем пираты, вероятно, не видели, потому что за много веков смысл успел
позабыться. Для кого это кое-что предназначалось?

Ответ:
Для Харона.

Комментарий:
Разумеется, пираты не верили в древнегреческих богов, но традиция класть
медяки на глаза усопшему сохранилась.

Источник:
Р.Л. Стивенсон. Остров сокровищ. http://flibusta.is/b/401616/read

Автор:
Мишель Матвеев (Санкт-Петербург)

Вопрос 27:
На одном сайте предлагают переименовать государство с населением около
10 миллионов человек в соответствии с веяниями времени. Название
государства при этом удлинится почти в три раза. А как это государство
называется сейчас?

Ответ:
Нигер.

Комментарий:
Предлагают переименовать в "Афроамериканец". А в Нигерии жителей на
порядок больше, да и название в три раза длиннее не получится.

Источник:
http://www.anekdot.ru/id/149315/

Автор:
Мишель Матвеев (Санкт-Петербург)

Вопрос 28:
При пересечении границ и осаде городов в римской армии полагалось
призвать перейти на сторону Рима местных ИХ, пообещав надлежащее
уважение. Назовите ИХ.

Ответ:
Боги.

Комментарий:
В те времена у городов и местностей обычно были свои боги. Римляне
призывали их перейти на свою сторону, обещая надлежащее почтение.
Впоследствии многие такие боги действительно пополняли римский пантеон.

Источник:
http://magazines.russ.ru/october/2007/7/va5.html

Автор:
Мишель Матвеев (Санкт-Петербург)

Вопрос 29:
Римляне при вторжении в чужую местность призывали местных богов перейти
на их сторону. Однако они предупреждали богов, желающих человеческих
жертвоприношений, что смогут предложить только ИХ. ОНИ появились
благодаря погребальному обряду. Назовите ИХ двумя словами.

Ответ:
Гладиаторские бои.

Комментарий:
Человеческие жертвоприношения для римлян были неприемлемы, однако
гладиаторов, погибших во время игр, соглашались посвятить богам, что
можно было счесть некоторым компромиссом. Гладиаторские бои возникли
благодаря погребальному обряду этрусков, в котором мужчины с мечами
сражались около могилы.

Источник:
   1. http://magazines.russ.ru/october/2007/7/va5.html
   2. http://ru.wikipedia.org/wiki/Гладиатор

Автор:
Мишель Матвеев (Санкт-Петербург)

Вопрос 30:
Когда Луиш Фигу приехал в составе "Реала" на стадион "Ноу Камп",
болельщики "Барселоны" выкрикивали ему просьбы одолжить... Что?

Ответ:
Тридцать сребреников.

Зачет:
Сребреники.

Комментарий:
Фигу, долгое время выступавший за "Барселону" и затем перешедший в
состав ее извечного противника, воспринимался болельщиками как
предатель.

Источник:
http://cup2006.lenta.ru/stars/figo/

Автор:
Мишель Матвеев (Санкт-Петербург)

Вопрос 31:
Размышляя о проблемах многонационального российского государства,
журналист Быков выражает надежду, что в будущем появится "единая нация,
сплотившаяся не по [ПРОПУСК], а по [ПРОПУСК] признакам". Заполните
пропуски словами, отличающимися на одну букву.

Ответ:
Этническим, этическим.

Источник:
http://www.russ.ru/columns/bikov/20040907.html

Автор:
Мишель Матвеев (Санкт-Петербург)

Вопрос 32:
Согласно "Всеобщей истории, обработанной "Сатириконом"", славяне
отличались большим гостеприимством. Гостю отводилось лучшее место в доме
и отдавались лучшие куски. Но какой ИКС потом получал гость, историкам
неизвестно. Назовите ИКС.

Ответ:
Счет.

Источник:
Всеобщая история, обработанная "Сатириконом".
http://flibusta.is/b/159772/read

Автор:
Мишель Матвеев (Санкт-Петербург)

Вопрос 33:
Согласно популярному в американском обществе замечанию, Микки-Маус
отличается от обычной полевой мыши только ЭТИМ. Википедия отмечает, что
ЭТО существует с глубокой древности, хотя название появилось только в XX
веке. Назовите ЭТО максимально коротко.

Ответ:
PR.

Комментарий:
Микки-Маус - самая обычная мышь, просто хорошо пропиаренная.

Источник:
   1. К.В. Душенко. Мысли, афоризмы и шутки знаменитых мужчин.
https://books.google.ru/books?id=PYifAAAAQBAJ&pg=PT296#v=onepage&q&f=false
   2. http://ru.wikipedia.org/wiki/Связи_с_общественностью

Автор:
Мишель Матвеев (Санкт-Петербург)

Вопрос 34:
Некто в Интернете утверждал, что у оптимистов больше страдает ноль, а у
пессимистов - девятка. Ему привели краткий аргумент, что может быть и
наоборот. Воспроизведите этот аргумент.

Ответ:
(:

Зачет:
(-:

Комментарий:
Человек утверждал, что оптимисты чаще используют улыбающийся смайлик :),
а пессимисты - грустный смайлик :(, поэтому у оптимистов быстрее
стирается символ 0, расположенный на той же клавише, что и закрывающая
скобка, а у пессимистов, соответственно, 9.

Источник:
http://bash.im/quote/397384

Автор:
Мишель Матвеев (Санкт-Петербург)

Вопрос 35:
Почти все люди, о которых идет речь в книге "Сто великих ИКСОВ", -
русские. Заканчивается она лауреатом Нобелевской премии. Назовите ИКСА
словом-палиндромом.

Ответ:
Казак.

Комментарий:
Лауреат - Шолохов, автор "Тихого Дона", основные персонажи которого -
казаки.

Источник:
А.В. Шишов. 100 великих казаков. http://flibusta.is/b/248784/read

Автор:
Мишель Матвеев (Санкт-Петербург)

Вопрос 36:
Персонаж Вудхауза, добравшись, наконец, в гости, на вопрос "Когда ты
приехал?" отвечает: "Только что разорвал ЕЕ". Назовите ЕЕ двумя словами.

Ответ:
Финишная ленточка.

Зачет:
Финишная лента.

Комментарий:
Как марафонец на финише дистанции. А наш турнир тоже завершен.

Источник:
П.Г. Вудхауз. Дживз уходит на каникулы. http://flibusta.is/b/148494/read

Автор:
Мишель Матвеев (Санкт-Петербург)

Тур:
3 тур

Дата:
09-Dec-2016

Редактор:
Алексей Полевой (Гомель) и Денис Рыбачук (Брест)

Инфо:
Редакторы благодарят за помощь в работе над пакетом команду "Ультиматум"
(Гомель), а также Яну Азриэль (Хайфа), Александра Кудрявцева (Николаев),
Николая Слюняева, Сергея Лобачёва, Ярослава Косарева (все - Нижний
Новгород), Тараса Вахрива (Тернополь), Наиля Фарукшина (Навои - Москва),
Сергея Терентьева (Санкт-Петербург) и Максима Мерзлякова (Воронеж).

Вопрос 1:
В семье одного блогера домашние вещи получают имена. Например, начав
читать пошаговую инструкцию по сборке на английском языке, столу дали
русское имя. Какое?

Ответ:
Степан.

Комментарий:
От английского "step one" [стэп уАн] - первый шаг.

Источник:
http://yapritopala.livejournal.com/192078.html

Автор:
Алексей Полевой (Гомель)

Вопрос 2:
Превознося ловкость своих соотечественников, Георг ШОлти говорил, что
быть венгром - это значит войти в дверь за вами, а выйти перед вами. В
какую дверь?

Ответ:
Вращающуюся.

Зачет:
Карусельную; револьверную; крутящуюся.

Источник:
Л. Паваротти, У. Райт. Мой мир http://flibusta.is/b/282947/read

Автор:
Денис Рыбачук (Брест)

Вопрос 3:
Персонаж Мигеля АстУриаса любит запрещенные азартные игры, поэтому носит
с собой дрессированного петуха. Во время игры он не нужен, но если
появляются полицейские, то петух ДЕЛАЕТ ЭТО. Если собака СДЕЛАЛА ЭТО, то
может понадобиться операция. Что мы заменили словами "ДЕЛАТЬ ЭТО"?

Ответ:
Глотать кости.

Зачет:
Проглатывать кости; заглатывать кости; съедать кости.

Комментарий:
Игральные кости незаметно сбрасываются под стол и обученный петух
принимает их за крупные съедобные зерна. Во втором случае имеются в виду
уже не игральные, а обычные кости, проглотив которые собака серьезно
рискует здоровьем.

Источник:
   1. М. Астуриас. Маисовые люди. http://flibusta.is/b/71203/read
   2. http://poisk-druga.ru/beginers/2602-chto-delat-esli-sobaka-proglotila-kost.html

Автор:
Алексей Полевой (Гомель)

Вопрос 4:
По легенде, пустыня началА наступать на плодородные земли после того,
как один мусульманин совершил покушение... На кого?

Ответ:
На сфинкса.

Комментарий:
Исламист обезобразил языческого сфинкса, и его родной город Гиза был
проклят.

Источник:
Г. Хэнкок, Р. Бьювэл. Загадка Сфинкса. http://flibusta.is/b/138425/read

Автор:
Денис Рыбачук (Брест)

Вопрос 5:
В вопросе мы сделали небольшое изменение.
   В 1912 году в Лувре был установлен памятник Чарльзу Роллсу. Что он
сделал первым из англичан?

Ответ:
Перелетел через Па-де-Кале на самолете.

Зачет:
По словам "перелетел" и "Па-де-Кале" или "Ла-Манш".

Комментарий:
Роллс был не только знаменитым автомобилестроителем, но и пионером
воздухоплавания. В городе Дувре, который находится на берегу Па-де-Кале,
установлен памятник летчику, который первым из англичан перелетел во
Францию.

Источник:
   1. http://enjoy-england.livejournal.com/15594.html
   2. https://doverhistorian.com/2014/02/21/charles-rolls-the-first-two-way-non-stop-english-channel-flight/

Автор:
Алексей Полевой (Гомель)

Вопрос 6:
Опальная княжна Тараканова искала поддержки даже среди высшего
духовенства. Назовите словом латинского происхождения то, из-за чего ее
послания некоторое время не могли быть пЕреданы адресатам.

Ответ:
Конклав.

Комментарий:
Княжна искала поддержку даже среди католического духовенства. Кардиналы
были заперты в помещении до того момента, пока не избрали нового Папу,
поэтому принять послания не могли. Слово "латинского" - небольшая
подсказка.

Источник:
Б.Т. Воробьёв. Книга тайн. http://flibusta.is/b/458611/read

Автор:
Денис Рыбачук (Брест)

Вопрос 7:
Концы кос у девочки, которая сидела перед озорным персонажем одного
романа, приобрели неестественный для волос цвет. Впоследствии волосы
девочки приходят в порядок, ведь все начинают пользоваться... Ответьте
двумя словами: чем?

Ответ:
Шариковыми ручками.

Комментарий:
Сидящий сзади герой-школьник нередко окунает косы девочки в чернильницу.
После того как всем говорят использовать шариковые ручки, эта
возможность у проказника пропадает.

Источник:
Э. Пру. Грехи аккордеона. http://flibusta.is/b/92109/read

Автор:
Алексей Полевой (Гомель)

Вопрос 8:
В одном произведении обитателю океана говорят, что тот не отличается
добротой, хоть у него и... Завершите реплику двумя словами,
начинающимися на соседние буквы алфавита.

Ответ:
Три сердца.

Комментарий:
У осьминога три сердца: одно (главное) гонит голубую кровь по всему
телу, а два других - жаберных - проталкивают кровь через жабры.

Источник:
Мультфильм "В поисках Дори" (2016), реж. Эндрю Стэнтон, Энгус МакЛэйн.

Автор:
Денис Рыбачук (Брест)

Вопрос 9:
Из-за ИКСА казалось, что голова персонажа РомЕна ГарИ лежит на блюде.
Сам персонаж тоже был ИКСОМ. Назовите ИКС двумя словами.

Ответ:
Белый воротничок.

Зачет:
Белый воротник.

Комментарий:
Персонаж является чиновником. "Белый воротничок" - обозначение, принятое
для наемного работника, занимающегося умственным трудом.

Источник:
Р. Гари. Большая барахолка. http://flibusta.is/b/349668/read

Автор:
Денис Рыбачук (Брест)

Вопрос 10:
По легенде, даже спустя некоторое время ОНА всё еще обличала царя и его
сожительницу. Назовите ЕЕ тремя словами.

Ответ:
Голова Иоанна Крестителя.

Комментарий:
По легенде, даже будучи отрубленной, голова святого продолжала свое
святое дело - обличала Ирода и Иродиаду.

Источник:
http://ru.wikipedia.org/wiki/Глава_Иоанна_Предтечи

Автор:
Алексей Полевой (Гомель)

Вопрос 11:
Словом "ИКС" мы заменили два слова.
   Героиня Николь КрАусс очень страдает, и вскоре появляется целая стая
ИКСОВ. Чтобы появился ИКС, нужно одиннадцать... Чего?

Ответ:
Сгибов [бумаги].

Зачет:
Складываний [бумаги].

Комментарий:
Если загадать желание (например, выздороветь) и сделать тысячу бумажных
журавликов (ИКСОВ), то желание должно исполниться.

Источник:
Н. Краусс. Большой дом.
http://www.loveread.ec/read_book.php?id=17854&p=52

Автор:
Алексей Полевой (Гомель)

Вопрос 12:
Персонаж одного произведения отказывается от достижений цивилизации, но
собирается использовать ручную дрель, чтобы с ее помощью... Что делать?

Ответ:
Добывать огонь.

Комментарий:
Необходимого трения для воспламенения материалов легче достигнуть с
помощью дрели.

Источник:
О. Хаксли. О дивный новый мир. http://flibusta.is/b/470368/read

Автор:
Денис Рыбачук (Брест)

Вопрос 13:
Говоря о том, что в начале XX века ОНИ поднялись, Дмитрий Лихачёв
замечает, что экстравагантная Мария ПунИ носила золотой браслет.
Назовите ИХ.

Ответ:
Подолы платьев.

Зачет:
Подолы; платья; юбки.

Комментарий:
Мария, знакомая матери Дмитрия, носила браслет на щиколотке. Заметить
это Дмитрий смог, только когда мода и раскрепощение нравов позволили
"поднять" высоту подолов.

Источник:
Д.С. Лихачёв. Я вспоминаю. http://flibusta.is/b/243699/read

Автор:
Алексей Полевой (Гомель)

Вопрос 14:
Живя на Капри, Максим Горький одевался ярко. По словам Юрия Анненкова,
это было протестом. Ответьте двухкоренным словом: против кого?

Ответ:
Чернорубашечников.

Комментарий:
Чернорубашечники - вооруженные отряды Национальной фашистской партии,
появившиеся в Италии после Первой мировой.

Источник:
Ю.П. Анненков. Дневники моих встреч. http://flibusta.is/b/270687/read

Автор:
Денис Рыбачук (Брест)

Вопрос 15:
Альфред БушЕ купил в Париже участок земли, в центре которого поставил
круглый павильон с треугольной крышей. В начале XX века там открылся
комплекс из множества маленьких студий, в которых трудились художники.
Какое короткое название получил этот комплекс?

Ответ:
"Улей".

Комментарий:
Художники, словно пчелы, кропотливо трудились в маленьких студиях,
напоминающих соты, а вид здания напоминал улей.

Источник:
http://ru.wikipedia.org/wiki/Улей_(Париж)

Автор:
Денис Рыбачук (Брест)

Вопрос 16:
В одной книге говорится, что некоторые змеи питаются термитами. Чтобы
защитить ценную вещь от термитов, богатые жители Индии в Средние века в
качестве ЕЕ использовали кусочек змеиной кожи. Назовите ЕЕ.

Ответ:
Закладка.

Комментарий:
Змеиная кожа отпугивает термитов, так как змеи являются их естественными
врагами. Термиты едят не только дерево, но и бумагу. Богатые люди
защищали книги, которые в то время считались большой ценностью.

Источник:
   1. И.А. Халифман. Шмели и термиты. - М.: Детская литература, 1972.
   2. Документальный сериал "Сокровища Инда" (Treasures of the Indus)
(BBC, 2015), 2-я серия.

Автор:
Денис Рыбачук (Брест)

Вопрос 17:
Кто просил издателя Курта Вольфа, чтобы насекомое не рисовали?

Ответ:
[Франц] Кафка.

Комментарий:
Писатель был против трактовки иллюстраторами его книги "Превращение",
так сказать, в лоб. Сам Кафка предлагал, чтобы на обложке была
изображена открытая в темноту дверь комнаты.

Источник:
https://tjournal.ru/11033-perevod-prevrashenie-kafki-100-mislei-na-100-let

Автор:
Денис Рыбачук (Брест)

Вопрос 18:
Поль ЗЮмтор пишет, что в XVII веке среди голландской читающей публики
были популярны ОНИ. Отношение к литературе ОНИ имели разве что словом в
названии, но ценились за дотошное изображение реальности и полезные
сведения из коммерции и географии. Назовите ИХ.

Ответ:
Судовые журналы.

Зачет:
Вахтенные журналы; бортовые журналы.

Комментарий:
На XVII век приходится расцвет нидерландского мореходства.

Источник:
П. Зюмтор. Повседневная жизнь Голландии во времена Рембрандта.
http://flibusta.is/b/297902/read

Автор:
Денис Рыбачук (Брест)

Вопрос 19:
Словами "ДЕЛАЛ ЭТО" мы заменили несколько слов.
   В школьном возрасте Николай Заболоцкий выглядел очень молодо. Чтобы
попасть в кино, мальчик ДЕЛАЛ ЭТО, а потом шел вместе со старшим
товарищем. Кто, чтобы доказать, что не ДЕЛАЛ ЭТО, подробно описывает
день, когда покинул столицу?

Ответ:
[Александр] Керенский.

Комментарий:
Чтобы казаться взрослее, Николай надевал женское платье и шел в кино с
другом. Керенский подчеркивал, что история про женское платье является
выдумкой.

Источник:
   1. Литературная матрица. Учебник, написанный писателями. Том 2.
http://flibusta.is/b/219350/read
   2. А.Ф. Керенский. Гатчина. http://flibusta.is/b/103624/read

Автор:
Алексей Полевой (Гомель), Денис Рыбачук (Брест)

Вопрос 20:
Чтобы предотвратить угрозу государству, которая возникла из-за недавно
появившихся лис, на ночь ИХ стали помещать в клетки. Назовите ИХ двумя
словами.

Ответ:
ВОроны Тауэра.

Зачет:
Тауэрские вОроны.

Комментарий:
Известно пророчество, что когда в Тауэре не останется воронов -
королевство падёт. В Тауэре, кстати, проживают еще и дикие лисы, которые
по ночам охотятся на птиц. Чтобы защитить воронов, их на ночь запирают в
клетки.

Источник:
http://www.bbc.com/russian/society/2015/09/150902_vert_cul_dickens_raven_tale

Автор:
Алексей Полевой (Гомель), Денис Рыбачук (Брест)

Вопрос 21:
Жители одного античного города приняли за ИКСА апостола Павла, когда тот
объявил, что принес весть о боге. Военный корабль "ИКС" однажды был
замаскирован под торговое судно, чтобы неожиданно напасть на вражеский
фрегат. Назовите ИКСА.

Ответ:
Меркурий.

Зачет:
Гермес.

Комментарий:
Павел принес весть, поэтому был принят язычниками за бога-вестника. Бриг
"Меркурий", символично замаскированный под торговое судно (ведь Меркурий
- покровитель торговли), задолго до фрегата "Меркурий" тоже совершил
подвиг, внезапной атакой захватив шведское судно.

Источник:
   1. И.С. Свенцицкая. Судьбы апостолов. Мифы и реальность. - М.: Вече,
2006. - С. 33.
   2. http://a-sharkov.livejournal.com/339926.html

Автор:
Денис Рыбачук (Брест)

Вопрос 22:
В качестве одного из поводов для сомнений приводят то, что имя "Аарон"
на надгробии написано с двумя буквами "а", а не с одной, как писал ОН
сам. Назовите ЕГО.

Ответ:
[Элвис Арон] Пресли.

Зачет:
Элвис.

Комментарий:
Поклонники верят, что король - жив, и ищут различные доказательства
того, что в могиле не он. Хотя бы по написанию второго имени певца.

Источник:
Я. Шерер. Мой внутренний Элвис. http://flibusta.is/b/360997/read

Автор:
Алексей Полевой (Гомель)

Вопрос 23:
Чтобы спасти экипаж затонувшей подводной лодки, в качестве первой помощи
с поверхности в нее опускают шланг. Какими двумя словами назвал эту
процедуру Джозеф Горз?

Ответ:
Искусственное дыхание.

Комментарий:
По шлангу на подлодку подается воздух. Искусственное дыхание
используется при первой помощи в различных экстренных ситуациях, в том
числе и утоплениях.

Источник:
Дж. Горз. Подъем затонувших кораблей. http://flibusta.is/b/97401/read

Автор:
Денис Рыбачук (Брест)

Вопрос 24:
Однажды знаменитая Грейс Келли оказалось в магазине без сопровождения.
Донести сумки Келли попросила одного из НИХ, и история попала в прессу.
Назовите ИХ несклоняемым словом.

Ответ:
Папарацци.

Комментарий:
Папарацци постоянно преследовали Грейс Келли, ведь та была не только
звездой Голливуда, но еще и супругой правящего монарха.

Источник:
Дж. Робинсон. Принцесса Монако. http://flibusta.is/b/369626/read

Автор:
Денис Рыбачук (Брест)

Вопрос 25:
Экипажи средиземноморских пиратских кораблей были весьма пестрыми по
составу. Артуро Перес-Реверте называет одно из таких судов плавающей ЕЮ.
Назовите ЕЕ.

Ответ:
Вавилонская башня.

Комментарий:
Экипаж был разношерстным по этническому составу, и его члены для общения
использовали множество разных языков.

Источник:
А. Перес-Реверте. Корсары Леванта. http://flibusta.is/b/369337/read

Автор:
Денис Рыбачук (Брест)

Вопрос 26:
Ученый Александр Ферсман был тучным, но весьма энергичным и
непредсказуемым человеком. Какое прозвище из двух слов дали ему друзья?

Ответ:
Шаровая молния.

Комментарий:
Шаровая молния очень энергичная и непредсказуемая, а слово "тучный" -
подсказка.

Источник:
Документальный фильм "Александр Ферсман. Властелин камней" из цикла
"Генералы в штатском".
http://tvkultura.ru/video/show/brand_id/20939/episode_id/167776/

Автор:
Денис Рыбачук (Брест)

Вопрос 27:
Персонаж РобЕра МЕрля является ИМ, поэтому в момент ядерного катаклизма
оказывается в безопасном месте. Правилом ИХ во Франции является
"терруАр". Назовите ИХ.

Ответ:
Виноделы.

Комментарий:
Винный погреб, находящийся глубоко под землей, выдерживает световой и
тепловой удары. Правилом виноделов является терруар, означающий,
например, что вино "Бордо" может быть произведено только на
соответствующей территории. Известный фильм "Правила виноделов" совсем
не об этом, но мог помочь ответить на этот вопрос.

Источник:
   1. Р. Мерль. Мальвиль. http://flibusta.is/b/36925/read
   2. http://ru.wikipedia.org/wiki/Терруар

Автор:
Алексей Полевой (Гомель)

Вопрос 28:
Слова "ПЕРВЫЕ" и "ВТОРЫЕ" - замены.
   В одной передаче говорится, что бОльшая часть драгоценностей,
найденных на месте поселений кочевников-гуннов, предназначались для
украшения ПЕРВЫХ, а не для ВТОРЫХ. На печати совета одного города
изображена ВТОРАЯ на ПЕРВОЙ. Назовите этот город.

Ответ:
Ковентри.

Комментарий:
Кочевники гунны больше украшали своих лошадей (ПЕРВЫЕ), нежели женщин
(ВТОРЫЕ). Печать создана по мотивам городской легенды о леди Годиве.

Источник:
   1. Документальный сериал "Терри Джонс и варвары" (Terry Jones'
Barbarians) (BBC, 2006).
   2. http://ru.wikipedia.org/wiki/Леди_Годива

Автор:
Денис Рыбачук (Брест)

Вопрос 29:
При блокаде Акры крестоносцами ОНИ передавали письма от осажденных
союзникам. По-словенски ОН - potaplja&#269; [потапляч]. Назовите ЕГО.

Ответ:
Ныряльщик.

Зачет:
Водолаз.

Комментарий:
Город был полностью обложен войсками, но это была не осада, а именно
блокада, ведь город имел выход к морю. Единственным способом подать
весточку было проплыть от стен города под водой и выйти на сушу вдалеке
от войск.

Источник:
   1. Дж. Горз. Подъем затонувших кораблей.
http://flibusta.is/b/97401/read
   2. https://translate.google.ru/#sl/ru/potaplja%C4%8D

Автор:
Денис Рыбачук (Брест)

Вопрос 30:
ЕЕ отец поддерживал луддитов, которые были противниками машин, но после
развода родителей ОНА попала под влияние матери. Назовите ЕЕ имя.

Ответ:
Ада [Лавлейс].

Зачет:
Августа; Огаста; то же с фамилией "Байрон".

Комментарий:
Анна Изабелла Байрон после расставания возненавидела своего мужа Джорджа
Гордона Байрона и, чтобы дочь не пошла по стопам отца, настаивала на
изучении точных наук. Ада всячески способствовала техническому прогрессу
и, в частности, занималась созданием программ для жаккардовых ткацких
машин.

Источник:
Документальный фильм "Ада Лавлейс: первая леди программирования"
(Calculating Ada: The Countess of Computing) (BBC, 2015).

Автор:
Денис Рыбачук (Брест)

Вопрос 31:
   <раздатка>
   The Repository of ______ ______
   </раздатка>
   На одном сайте есть раздел, посвященный событиям, которых в
действительности не было. Цель раздела - посмеяться над доверчивыми
читателями и поощрить их проверять неправдоподобные утверждения. Его
англоязычное название заканчивается двумя словами, начинающимися на одну
и ту же букву. Напишите эту букву.

Ответ:
L.

Комментарий:
На сайте snopes.com [снОупс точка ком], посвященном городским легендам,
имеется раздел "The Repository of Lost Legends" [зе репОзитори оф лост
лЭджендс] - Хранилище утраченных легенд, содержащий обсуждения
выдуманных легенд, в том числе для того, чтобы потроллить читателей,
которые принимают всё на веру. Акронимом названия раздела является слово
"TROLL", понятное и без перевода.

Источник:
   1. http://ru.wikipedia.org/wiki/Фиктивная_запись
   2. http://www.snopes.com/lost/lost.asp

Автор:
Алексей Полевой (Гомель)

Вопрос 32:
Какой лондонский объект, согласно блогу enjoy-england [энджОй Ингланд],
соответствует одному из своих названий меньше чем наполовину?

Ответ:
Метрополитен.

Зачет:
Метро; underground; tube; subway.

Комментарий:
Лондонское метро называется андерграунд, или подземка. Интересно, что
около 60% протяженности всех линий находятся над землей.

Источник:
http://enjoy-england.livejournal.com/7297.html

Автор:
Алексей Полевой (Гомель)

Вопрос 33:
Путешествуя по разным странам, Рич МакКор создает своеобразные
фотоколлАжи. Так, на снимке, сделанном в Голландии, появляется часть
партии... Во что?

Ответ:
В крестики-нолики.

Комментарий:
Лопасти ветряной мельницы образовали крестик, а бумажные нолики на
фотоколлаж добавил МакКор.

Источник:
http://www.fresher.ru/2015/10/25/xudozhnik-kreativno-transformiruet-znamenitye-dostoprimechatelnosti/

Автор:
Денис Рыбачук (Брест)

Вопрос 34:
Героиня одного романа по имени Мэри, узнав о социальной проблеме,
спросила, "почему бы не воспользоваться прачечной", но вскоре ей уже
приписывали знаменитую фразу. Напишите слово, которым эта фраза
оканчивается.

Ответ:
Пирожные.

Зачет:
БриОши.

Комментарий:
Цены на мыло выросли, а жители бедных кварталов никак не могут
пользоваться куда более дорогими прачечными. Вскоре Мэри, допустившей
такую глупость, как и Марии-Антуанетте, приписывают фразу о хлебе и
пирожных.

Источник:
   1. М. Спарк. Мисс Джин Броди в расцвете лет.
http://flibusta.is/b/204144/read
   2. http://ru.wikipedia.org/wiki/Если_у_них_нет_хлеба,_пусть_едят_пирожные!

Автор:
Алексей Полевой (Гомель)

Вопрос 35:
Критик пишет, что в исторической пьесе Корнеля слишком много болтают...
Кто?

Ответ:
Спартанцы.

Зачет:
Лакедемонцы; жители Спарты.

Комментарий:
В те времена модными были пьесы с античным сюжетом. Спартанцы славились
своей лаконичностью, но Корнелю, возможно, платили за количество слов.

Источник:
Ф. Мориак. Жизнь Жана Расина.

Автор:
Денис Рыбачук (Брест)

Вопрос 36:
В одной из постановок на сцену выходил слуга в тяжелых башмаках.
Концовку какой пьесы мы описали?

Ответ:
"Вишневый сад".

Комментарий:
В тяжелых башмаках выходил забытый Фирс, а их стук символизировал стук
топора.

Источник:
А.С. Демидова. Бегущая строка памяти. http://flibusta.is/b/188480/read

Автор:
Денис Рыбачук (Брест)

Тур:
4 тур

Дата:
17-Feb-2017

Редактор:
Ярослав Косарев, Сергей Лобачёв (оба - Нижний Новгород), Сергей
Терентьев (Санкт-Петербург)

Инфо:
Редакторы благодарят за тестирование вопросов и ценные замечания Антона
Волосатова, Екатерину Лобкову, Максима Мерзлякова, Дмитрия Ожигова,
Николая Слюняева, Игоря Тюнькина, Наиля Фарукшина, Татьяну Фёдорову.

Вопрос 1:
Изначально очередность планировалась другая, но в таком случае ПЕРВОМУ
пришлось бы перелезать через ВТОРОГО, что, как показали тренировки, было
очень неудобно. Назовите ПЕРВОГО или ВТОРОГО.

Ответ:
[Базз] Олдрин.

Зачет:
[Нил] Армстронг.

Комментарий:
Олдрин должен был выходить из космического корабля первым, но тогда ему
пришлось бы перелезать через сидящего у выхода Армстронга. На тренировке
Олдрин чуть не разнес корабль подобным маневром, и очередность изменили.

Источник:
Р.Е. Зинзер. FAQ: Америка.

Автор:
Сергей Терентьев (Санкт-Петербург)

Вопрос 2:
Для борьбы с НИМ предлагают использовать огромную электродинамическую
сеть, которая позволила бы замедлить ЕГО и привести к падению. Назовите
ЕГО двумя словами.

Ответ:
Космический мусор.

Комментарий:
Сеть должна замедлить космический мусор, что приведет к его падению в
атмосферу, в которой он сгорит.

Источник:
https://hi-news.ru/space/sem-sposobov-borby-s-kosmicheskim-musorom.html

Автор:
Сергей Терентьев (Санкт-Петербург)

Вопрос 3:
Можно сказать, что жестУно - это ОНО для глухонемых. Назовите ЕГО.

Ответ:
Эсперанто.

Комментарий:
Глухонемые разных стран используют свои собственные языки жестов и
потому плохо понимают зарубежных товарищей. В попытке исправить эту
ситуацию был создан международный язык жестов - жестуно.

Источник:
http://en.wikipedia.org/wiki/International_Sign

Автор:
Сергей Лобачёв (Нижний Новгород)

Вопрос 4:
Натурщица Дина ВернИ начала позировать скульпторам еще в юности. На
многих работах АристИда МайОля у нее сосредоточенный взгляд и
наклоненная вниз голова. Дело в том, что во время сеансов позирования
Дина делала... Закончите предыдущее предложение одним словом.

Ответ:
Уроки.

Комментарий:
Верни начала позировать Майолю будучи школьницей. Свободного времени для
позирования у нее, видимо, было немного. Поэтому скульптор приглашал ее
делать уроки у него в мастерской.

Источник:
В.Я. Вульф, С.А. Чеботарь. "Звезды", покорившие миллионы сердец.
http://flibusta.is/b/372134/read

Автор:
Ярослав Косарев (Нижний Новгород)

Вопрос 5:
Героиня стихотворения Марины Цветаевой говорит обольстительному
персонажу, что его имя ей незнакомо. Назовите этого персонажа.

Ответ:
[Джакомо] Казанова.

Комментарий:
Хотя в имени ДжАкомо ударение ставится на первый слог, его часто ставят
на второй - ДжакОмо. Слова "незнакомо" и "ДжакОмо" рифмуются.

Источник:
М.И. Цветаева. Феникс.
http://tsvetaeva.lit-info.ru/tsvetaeva/piesy/feniks/feniks-3.htm

Автор:
Сергей Терентьев (Санкт-Петербург)

Вопрос 6:
Одна организация считает, что проблемам Антарктиды уделяют недостаточно
внимания. Поэтому она привлекла к своей рекламной кампании спортсменов,
а слоган кампании предлагал добавить ЕГО. Назовите ЕГО.

Ответ:
Олимпийское кольцо.

Зачет:
Шестое кольцо; кольцо.

Комментарий:
На олимпийском флаге кольца символизируют континенты, однако для
Антарктиды кольца нет. По мнению создателей рекламной кампании, этот
слоган поможет привлечь внимание к проблеме глобального потепления,
затрагивающего Антарктиду.

Источник:
http://www.addthering.com/

Автор:
Сергей Лобачёв (Нижний Новгород)

Вопрос 7:
В церкви Тёбю есть фреска на неканонический сюжет, выполненная мастером
АльбЕртусом. На ней изображен предмет непривычной нам прямоугольной
формы, рядом с которым сидят двое. Назовите произведение 1957 года,
создатель которого, как считается, вдохновлялся этой фреской.

Ответ:
"Седьмая печать".

Комментарий:
Церковь расположена в Швеции. Изображенные на ней человек и смерть сидят
с одной стороны, и если не знать канонов того времени, то может
показаться, что один из них подсказывает другому. По одной из версий,
эта фреска подала Ингмару Бергману идею для фильма.

Источник:
http://ru.wikipedia.org/wiki/Смерть,_играющая_в_шахматы

Автор:
Сергей Терентьев (Санкт-Петербург)

Вопрос 8:
В этом вопросе словом "ИКС" заменены два слова.
   В восточной традиции ИКС ассоциируется с агрессией, а потому го -
одна из немногих игр, в которых ПРОПУСК. Восстановите пропуск, состоящий
из трех слов.

Ответ:
Черные ходят первыми.

Зачет:
Ответы из трех слов, подходящие по смыслу, например: "Черные начинают
игру".

Комментарий:
ИКС - черный цвет. В традиционной японской настольной игре человек,
играющий черными камнями, делает свой ход первым.

Источник:
"Вокруг света", 2016, N 12.

Автор:
Сергей Лобачёв (Нижний Новгород)

Вопрос 9:
Граф РОджер МЕннерс был очень образован, много путешествовал и бывал в
Дании. Журнал "Вокруг света" отмечает, что после смерти графа не
появилось ни одной... Ответьте двумя словами: чего?

Ответ:
Пьесы Шекспира.

Зачет:
Книги Шекспира; хроники Шекспира; комедии Шекспира; трагедии Шекспира.

Комментарий:
По одной из версий, Роджер Меннерс был литературным негром Шекспира и
автором большинства его пьес.

Источник:
   1. "Вокруг света", 2016, N 7. - С. 69.
   2. http://ru.wikipedia.org/wiki/Роджер_Меннерс,_5-й_граф_Ратленд

Автор:
Сергей Лобачёв (Нижний Новгород)

Вопрос 10:
Паломники, опасавшиеся грабителей, могли хранить свои деньги у
тамплиеров. Гарантией выдачи денег служили ОНИ на специальной расписке.
Назовите ИХ двумя словами, начинающимися на соседние буквы алфавита.

Ответ:
Отпечатки пальцев.

Комментарий:
Человек, который хотел воспользоваться услугой тамплиеров, менял свои
деньги на расписку с отпечатками собственных пальцев. Чтобы получить
деньги назад, человек должен был пройти процедуру сверки отпечатков
пальцев.

Источник:
"Наука и жизнь", 2016, N 8. - С. 72.

Автор:
Сергей Лобачёв (Нижний Новгород)

Вопрос 11:
На одной фотографии на НИХ изображены знаки "воспроизведение", "пауза" и
"стоп", хотя функция одной из НИХ не совсем соответствует паузе.
Назовите любую из НИХ двумя словами.

Ответ:
Педаль газа.

Зачет:
Педаль тормоза; педаль сцепления; автомобильная педаль; педаль
автомобиля.

Комментарий:
Функции знаков воспроизведения похожи на функции педалей автомобиля.

Источник:
http://pikabu.ru/story/pamyatka_voditelyu_2104347

Автор:
Сергей Терентьев (Санкт-Петербург)

Вопрос 12:
Перед своей свадьбой в 1794 году КретьЕн ГийОм де МальзЕрб оступился и
пошутил, что это верный знак того, что ему следовало остаться дома.
Какое слово мы заменили в этом вопросе?

Ответ:
Казнью.

Зачет:
Казнь.

Комментарий:
Мальзерб - французский политик и адвокат, который защищал на суде короля
Людовика XVI, за что и попал на эшафот.

Источник:
http://borisakunin.livejournal.com/91231.html

Автор:
Ярослав Косарев (Нижний Новгород)

Вопрос 13:
Рассказывают, что одну из своих пьес Нил Саймон специально написал очень
плохой. Причиной такого решения стало то, что незадолго до постановки
пьесы он СДЕЛАЛ ЭТО. Ответьте одним словом: что именно сделал?

Ответ:
Развелся.

Комментарий:
Нил Саймон обещал своей жене доходы от первой пьесы, написанной после
развода.

Источник:
http://en.wikipedia.org/wiki/Fools_(play)

Автор:
Сергей Лобачёв (Нижний Новгород)

Вопрос 14:
Леди бест - персонаж вселенной Marvel [мАрвел]. Она устраивала пожары в
банках и грабила их, пользуясь тем, что полиция не могла ей помешать.
Какие две буквы мы пропустили в этом вопросе?

Ответ:
Ас.

Комментарий:
В вопросе имеется в виду Леди Асбест. Асбест известен своей
огнестойкостью.

Источник:
   1. http://www.marvel.com/universe/Asbestos_Lady
   2. http://ru.wikipedia.org/wiki/Асбест

Автор:
Сергей Лобачёв (Нижний Новгород)

Вопрос 15:
Первого апреля разработчики "World of Warcraft" [ворлд оф варкрАфт]
анонсировали новый артефакт - АЛЬФУ, позволяющую скрывать статистику
игрока. Хирурги иногда используют АЛЬФУ для защиты от инфракрасного
излучения. Назовите АЛЬФУ тремя словами.

Ответ:
Шапочка из фольги.

Комментарий:
Считается, что шапочка из фольги может защитить от вредных излучений и
влияний. Но у нее действительно есть практическое применение - такая
шапочка снижает тепловую нагрузку на голову врачей во время некоторых
операций.

Источник:
http://ru.wikipedia.org/wiki/Шапочка_из_фольги

Автор:
Сергей Терентьев (Санкт-Петербург)

Вопрос 16:
В правительстве Гувера полагали, что джентльмены должны доверять друг
другу. Во многом из-за этих взглядов правительство отказалось
финансировать "Сфинкс", американский аналог ЕЕ. Назовите ЕЕ одним
словом.

Ответ:
"Энигма".

Зачет:
"Enigma".

Комментарий:
В 1915 году американец Эдвард Хепберн разработал автоматическую
шифровальную машину, которую сам называл "сфинксом радиосвязи". Из-за
политики открытости власти США отказались спонсировать проект, что
привело к банкротству изобретателя. По случайному совпадению названия
"Сфинкс" и "Энигма" связаны с загадками.

Источник:
   1. С. Сингх. Книга шифров. Тайная история шифров и их расшифровки.
http://flibusta.is/b/419363/read
   2. http://www.aif.ru/dontknows/file/chto_takoe_enigma

Автор:
Сергей Лобачёв (Нижний Новгород)

Вопрос 17:
Современники считали этого англичанина еще бОльшим врагом, чем
Наполеона. В частности, он выступал против лечения оспы и не видел
ничего плохого в рабстве и детоубийстве. Назовите его.

Ответ:
[Томас] Мальтус.

Комментарий:
Согласно теории Мальтуса, неконтролируемый рост населения в конце концов
приведет к массовому голоду. Упомянутые в вопросе меры контроля
населения Мальтус, видимо, считал необходимым злом.

Источник:
Роберт Хайлбронер. Философы от мира сего. Великие экономические
мыслители: их жизнь, эпоха и идеи. http://flibusta.is/b/294019/read

Автор:
Ярослав Косарев (Нижний Новгород)

Вопрос 18:
С началом Великой депрессии одно развлечение стало еще менее доступным,
что повысило популярность ЕГО. В ход шли старые водосточные трубы, шины
и бочки. Назовите ЕГО, использовав дефис.

Ответ:
Мини-гольф.

Комментарий:
Гольф всегда был недешевым удовольствием, а экономические проблемы
породили развитие мини-гольфа. Шины, бочки, а также водосточные трубы
использовались для создания препятствий.

Источник:
http://entertainment.howstuffworks.com/sports/golf/basics/who-built-first-miniature-golf-course.htm

Автор:
Сергей Лобачёв (Нижний Новгород)

Вопрос 19:
В былые времена участки исландских землевладельцев не имели формальных
границ. Варвара Лозенко пишет, что из-за этого исландцы иногда затевали
небольшие междоусобные войны, пытаясь выяснить, на чью территорию...
Закончите мысль двумя словами.

Ответ:
Выбросился кит.

Зачет:
По смыслу, с использованием двух слов.

Комментарий:
Мясо кита могло кормить исландскую семью на протяжении целого года.

Источник:
"Geo", 2015, N 10. - С. 101.

Автор:
Сергей Лобачёв (Нижний Новгород)

Вопрос 20:
Билл Брайсон был недоволен посещением Лувра - сперва ему пришлось долго
стоять в очереди на вход в музей, а позже "Мона Лиза" показалась ему
АЛЬФОЙ. В Бельгии выпускаются АЛЬФЫ, при использовании которых можно
почувствовать вкус шоколада. Назовите АЛЬФУ.

Ответ:
[Почтовая] марка.

Комментарий:
Так как в музее было много посетителей, Брайсон смог увидеть картину
только издалека. Бельгийская почта выпустила марки, клеевой состав
которых пах шоколадом.

Источник:
   1. Б. Брайсон. Путешествия по Европе.
http://flibusta.is/b/174128/read
   2. http://www.mk.ru/politics/world/2013/03/25/830700-v-belgii-vyipustili-vkusnyie-marki-s-shokoladom.html

Автор:
Ярослав Косарев (Нижний Новгород)

Вопрос 21:
Одним из прототипов для "НЕЕ" послужил "Шиповничек" известных братьев.
Назовите "ЕЕ" двумя словами.

Ответ:
"Спящая царевна".

Зачет:
"Спящая красавица".

Комментарий:
Братья Гримм написали сказку с названием "Шиповничек" на основе
распространенного в Европе сюжета о спящей красавице. На основе этой
сказки Жуковский написал балладу "Спящая царевна". В сказке братьев
Гримм девушка укололась шиповником, который, как можно понять по
названию, колюч.

Источник:
http://www.myshared.ru/slide/476143/

Автор:
Сергей Терентьев (Санкт-Петербург)

Вопрос 22:
Каждое из слов "АЛЬФА" и "БЕТА" в тексте вопроса заменяет другое слово.
   В XVIII веке английские женщины могли при помощи АЛЬФ
продемонстрировать свою политическую принадлежность. Рассказывая об
этом, Люси Уорсли пишет, что одной стороннице вигов от природы не
повезло, и из-за БЕТЫ ее относили к приверженцам тори. Назовите АЛЬФУ и
БЕТУ.

Ответ:
Мушка, родинка.

Комментарий:
В зависимости от того, на какой части лица наклеена мушка - на правой
или на левой, - ею обозначалась принадлежность к партии тори или к
партии вигов. У известной сторонницы вигов от рождения была родинка,
которая расположилась неудачно - не на "нужной" стороне лица.

Источник:
Л. Уорсли. Английский дом. Интимная история.
http://flibusta.is/b/449163/read

Автор:
Сергей Терентьев (Санкт-Петербург)

Вопрос 23:
Одним из первых в новом значении слово "ИКС" использовал юморист Дуглас
Кенни. Он считал, что люди могут сэкономить время, и собрал ИКСЫ
классических произведений и детективов. Назовите ИКС.

Ответ:
Спойлер.

Комментарий:
Спойлер - гаситель подъемной силы. Слово "спойлер" в значении
преждевременно раскрытой информации впервые использовал Дуглас Кенни,
который собрал большое число спойлеров из разных произведений, считая,
что людям надо поменьше смотреть фильмы и читать.

Источник:
http://www.mirf.ru/kino/spoiler-ne-nado-boyatsya

Автор:
Сергей Терентьев (Санкт-Петербург)

Вопрос 24:
По легенде, монахи ХугАрдена обучили сироту КлАаса секретам мастерства,
и впоследствии тот завещал использовать ПЕРВУЮ в качестве ВТОРОГО.
Назовите ПЕРВУЮ и ВТОРОЙ.

Ответ:
[Пивная] бочка, гроб.

Комментарий:
Хугарден - это бельгийская деревня, в которой с XIII века варят
одноименное пиво. Местные монахи подобрали Клааса еще ребенком, и всю
свою жизнь он посвятил пивоварению.

Источник:
   1. http://www.hoegaarden.com/ru/the-brand/
   2. http://ru.wikipedia.org/wiki/Hoegaarden

Автор:
Сергей Лобачёв (Нижний Новгород)

Вопрос 25:
Рассказывают, что ОНИ согласились сотрудничать, сказав: "Нет бОльших
патриотов, чем коренные американцы". Назовите ИХ точно.

Ответ:
[Индейцы] навахо.

Комментарий:
Во время Второй мировой войны индейцы навахо служили шифровальщиками
сообщений.

Источник:
С. Сингх. Книга шифров. Тайная история шифров и их расшифровки.
http://flibusta.is/b/419363/read

Автор:
Сергей Лобачёв (Нижний Новгород)

Вопрос 26:
В одной шутке СИри отказывается загружать альбом БАсты и СмОки Мо,
ссылаясь на один из НИХ. Назовите ИХ двумя словами.

Ответ:
Законы роботехники.

Зачет:
Законы робототехники; законы Азимова.

Комментарий:
Персональный помощник Сири ссылается на первый закон роботехники,
запрещающий роботам причинять человеку вред.

Источник:
(pic: 20160742.jpg)

Автор:
Сергей Лобачёв (Нижний Новгород)

Вопрос 27:
Стивен ВАйнберг пишет, что в Александрийском мусейОне, основанном
Птолемеем, ОНА засияла ярче своих сестер. Напишите ЕЕ имя.

Ответ:
Урания.

Комментарий:
Заведение "мусейон" получило свое название в честь муз. Основной наукой,
изучавшейся в мусейоне, была астрономия, которой соответствует муза
Урания.

Источник:
   1. С. Вайнберг. Объясняя мир. Истоки современной науки.
http://flibusta.is/b/430757/read
   2. http://ru.wikipedia.org/wiki/Александрийский_мусейон

Автор:
Сергей Лобачёв (Нижний Новгород)

Вопрос 28:
Астроном Алексей Пахомов образно пишет, что по вечерам на осеннем
светлом небе можно наблюдать ЕГО. Во Флоренции "ЕГО" можно наблюдать в
любое время года. Назовите ЕГО двумя словами.

Ответ:
Рождение Венеры.

Зачет:
"Рождение Венеры".

Комментарий:
В первом случае речь идет о планете, а во втором - о картине Боттичелли.

Источник:
   1. "Наука и жизнь", 2016, N 8. - С. 41.
   2. http://ru.wikipedia.org/wiki/Рождение_Венеры_(картина_Боттичелли)

Автор:
Сергей Лобачёв (Нижний Новгород)

Вопрос 29:
Для взятия этого вопроса вам нужно будет сосредоточиться.
   По-немецки ОН называется "brennpunkt" [бреннпункт], что можно
дословно перевести как "точка воспламенения". Назовите ЕГО.

Ответ:
Фокус.

Комментарий:
В оптике фокусом называют точку пересечения лучей, прошедших сквозь
собирающую систему. В предисловии к вопросу мы попросили вас
сосредоточиться, т.е. сфокусировать внимание.

Источник:
http://de.wikipedia.org/wiki/Brennpunkt_(Geometrie)

Автор:
Сергей Лобачёв (Нижний Новгород)

Вопрос 30:
В Северной Америке зима 2015 года выдалась особенно холодной. В
результате Уилл Гэд совершил первое в истории ЭТО. Назовите ЭТО точно.

Ответ:
Восхождение на Ниагарский водопад.

Зачет:
По смыслу, с упоминанием Ниагарского водопада.

Комментарий:
Это был не первый случай, когда замерз Ниагарский водопад, однако
восхождений до Уилла Гэда никто не совершал.

Источник:
http://www.redbull.com/ru/ru/adventure/stories/1331702511052/will-gadd-ice-climbing-niagara-falls

Автор:
Сергей Лобачёв (Нижний Новгород)

Вопрос 31:
Для одного развлечения сначала использовали конфеты "Life Savers" [лайф
сЭйверз], но производитель увеличил их размер. В этом развлечении также
используется ОНА. Назовите ЕЕ словом, пишущимся через дефис.

Ответ:
Кока-кола.

Зачет:
Пепси-кола.

Комментарий:
Для создания фонтана сейчас используется смесь "Кока-колы" и "Ментоса".
Раньше использовались другие конфеты, но производитель их увеличил, и
они перестали пролезать в горлышко бутылки.

Источник:
http://ru.wikipedia.org/wiki/Фонтан_из_кока-колы_и_Mentos

Автор:
Сергей Терентьев (Санкт-Петербург)

Вопрос 32:
Девять лет назад парижане устроили протест против подавления народных
выступлений в Лхасе. В результате действий демонстрантов ОНА была
прервана на некоторое время. Назовите ЕЕ тремя словами.

Ответ:
Эстафета олимпийского огня.

Комментарий:
Лхаса является столицей Тибетского автономного района, в котором прошли
беспорядки в связи с демонстрациями сторонников независимости района от
КНР. Протесты подхватили и в Париже, где в тот момент проходила эстафета
олимпийского огня, конечной точкой которой был Пекин.

Источник:
http://ria.ru/beijing2008/20080407/103925314.html

Автор:
Ярослав Косарев (Нижний Новгород)

Вопрос 33:
Чтобы выиграть дело, герой одного романа подарил судье корову. Однако
его противник оказался хитрее и подарил судье ТАКУЮ корову. Ответьте
словом с удвоенной согласной: какую именно?

Ответ:
Беременную.

Комментарий:
Взятка противника оказалась весомее.

Источник:
Д.С. Мережковский. Воскресшие боги, или Леонардо да Винчи.
http://flibusta.is/b/224676/read

Автор:
Ярослав Косарев (Нижний Новгород)

Вопрос 34:
В обязанности людей редкой профессии входят проверки магнитом,
балансировщиком и электронным микрОметром. Назовите двумя словами
предметы, которые таким образом проверяют.

Ответ:
Игральная кость.

Зачет:
Игральный кубик.

Комментарий:
Магнит должен подтвердить отсутствие металлических вставок в кости, а
при помощи микрометра и балансировщика работники убеждаются, что шансы
выпадения каждой грани одинаковые.

Источник:
http://entertainment.howstuffworks.com/how-do-they-test-casino-dice.htm

Автор:
Сергей Лобачёв (Нижний Новгород)

Вопрос 35:
При Медичи в искусстве преобладали библейские сюжеты. Эрик ВЕйнер пишет,
что меценатство Медичи было связано не только с любовью к искусству. С
чем в книге Вейнера сравниваются шедевры того времени?

Ответ:
С индульгенциями.

Комментарий:
Церковь в ответ на плату за шедевры искусства брала на себя заботы
насчет вечных мук Медичи.

Источник:
Э. Вейнер. География гениальности. Где и почему рождаются великие идеи.
http://flibusta.is/b/465171/read

Автор:
Ярослав Косарев (Нижний Новгород)

Вопрос 36:
В английском языке ОНА называется "tittle" [титл]. В этом вопросе ОНА
тоже есть. Ответьте тремя словами, что такое ОНА.

Ответ:
Точка над i.

Комментарий:
В тексте этого вопроса присутствует латинская буква i [ай] и,
соответственно, точка над i.

Источник:
http://blog.dictionary.com/tittle/

Автор:
Сергей Лобачёв (Нижний Новгород)

Тур:
5 тур

Дата:
10-Mar-2017

Редактор:
Александр Кудрявцев (Николаев)

Инфо:
Редактор благодарит за тестирование: Глеба Агапова, Яну Азриэль, Михаила
Барчука, Александра Битюкова, Александра Ваксмана, Тараса Вахрива,
Ростислава Гимчинского, Лидию Гуржий, Елену Даскал, Ирину Зубкову,
Андрея Кокуленко, Сергея Лобачёва, Бориса Моносова, Аркадия Руха,
Ярослава Смолянинова, Игоря Тюнькина, Серафима Шибанова и команду
"Проект "Геркулес"" (Кропивницкий).

Вопрос 1:
СтрабОн писал, что если бы ОН захотел встать с трона, то снес бы крышу.
Назовите ЕГО.

Ответ:
Зевс.

Зачет:
Зевс Олимпийский.

Комментарий:
Так древнегреческий историк и географ описывал одно из чудес света -
статую Зевса в Олимпии. Она была столь высока, что сидящий громовержец
почти касался головой потолка.

Источник:
Н.А. Ионина. 100 великих чудес света. http://flibusta.is/b/482765/read

Автор:
Александр Мудрый (Черновцы)

Вопрос 2:
Литовцы считали, что ОНИ - это муж и жена, которые развелись, но хотели
видеть свою дочь, и поэтому приходили к ней по очереди. Назовите ИХ.

Ответ:
Месяц, Солнце (в любом порядке).

Зачет:
Луна, Солнце (в любом порядке).

Комментарий:
Поэтому муж-Месяц навещает Землю ночью, а жена-Солнце - днем.

Источник:
http://ru.wikipedia.org/wiki/Сауле_(богиня)

Автор:
Юлия Лунёва (Каменское)

Вопрос 3:
Внимание, в вопросе слова "АЛЬФА" и "БЕТА" являются заменами.
   В русском переводе воспоминаний Гёте так описывается наряд дочери
деревенского пастора: "Пышная и короткая белая юбочка, почти до
щиколотки открывавшая очаровательнейшие ножки; узкий белый лиф и черный
тафтяной передник - полуАЛЬФА, полуБЕТА". Назовите автора произведения
"АЛЬФА-БЕТА".

Ответ:
[Александр Сергеевич] Пушкин.

Комментарий:
АЛЬФА - барышня, БЕТА - крестьянка. Так Гёте описывал наряд своей
возлюбленной Фридерики БриОн.

Источник:
   1. И.В. Гёте. Из моей жизни: Поэзия и правда.
http://flibusta.is/b/297311/read
   2. http://ru.wikipedia.org/wiki/Барышня-крестьянка

Автор:
Александр Мудрый (Черновцы)

Вопрос 4:
Внук УлИсса Гранта, полководца времен Гражданской войны, однажды
попросил СмитсОновский музей одолжить экспонат, некогда принадлежавший
его деду. Экспонатом собирались воспользоваться на юбилейном
мероприятии, для которого был изготовлен ОН высотой восемь футов.
Назовите ЕГО.

Ответ:
Торт.

Комментарий:
В Смитсоновском музее хранился парадный меч Улисса Гранта. На юбилее его
использовали, чтобы разрезать огромный торт.

Источник:
http://americanhistory.si.edu/blog/2011/10/general-grants-sword-takes-the-cake.html

Автор:
Александр Кудрявцев (Николаев)

Вопрос 5:
На рекламе американского тренажерного зала изображен упитанный мужчина,
стоящий между двумя буквами высотой в его рост, а ниже - тот же самый
мужчина несколько месяцев спустя. Напишите эти буквы.

Ответ:
F, T.

Зачет:
F, t.

Комментарий:
FAT [фэт] на этой рекламе превращается в FIT [фит].

Источник:
https://twitter.com/brilliant_ads/status/556116928092442624/

Автор:
Александр Кудрявцев (Николаев)

Вопрос 6:
Несколько десятилетий назад в Америке заметно уменьшился размер
пластмассовых детских кукол. Назовите двумя словами причину этого.

Ответ:
Нефтяной кризис.

Зачет:
Нефтяное эмбарго.

Комментарий:
Пластмассу, которая использовалась для производства игрушек, делали из
нефти. Нефтяной кризис 1970-х годов заставил производителей уменьшить
расходы.

Источник:
http://en.wikipedia.org/wiki/Action_figure#1960s.E2.80.931970s

Автор:
Александр Кудрявцев (Николаев)

Вопрос 7:
Программный набор DirectX [дИрект икс] и созданная на его базе приставка
Xbox [икс бокс] должны были утвердить американскую гегемонию в сфере
видеоигр, вытеснив из нее азиатских конкурентов. Первоначальное название
DirectX совпадало с названием известной программы. Что появилось
благодаря этой программе?

Ответ:
Атомное оружие.

Зачет:
Ядерное оружие; атомная бомба; ядерная бомба.

Комментарий:
Первоначальное название DirectX - Manhattan Project [мэнхЭтн прОджэкт].
На его раннем логотипе была изображена буква X [икс], стилизованная под
знак радиационной опасности. В современном логотипе ранний вполне
угадывается. "Манхэттенский проект" - кодовое название американской
программы по разработке ядерного оружия.

Источник:
http://en.wikipedia.org/wiki/DirectX#Logos

Автор:
Александр Кудрявцев (Николаев)

Вопрос 8:
Французские и бельгийские фермеры до сих пор нередко собирают на своих
полях так называемый ТАКОЙ урожай, который только во Франции составляет
900 тонн в год. Назовите изобретателя, в прозвище которого входит слово
"ТАКОЙ".

Ответ:
Энтони Эдвард Старк.

Зачет:
Тони Старк; Железный человек; Iron Man [чтецу: Айрон мэн].

Комментарий:
ТАКОЙ - железный. На полях Франции и Бельгии со времен мировых войн
осталось огромное количество неразорвавшихся снарядов и металлического
лома.

Источник:
   1. http://en.wikipedia.org/wiki/Iron_harvest
   2. http://ru.wikipedia.org/wiki/Железный_человек

Автор:
Александр Кудрявцев (Николаев)

Вопрос 9:
Содержащееся в НИХ вещество, которое является аналогом каннабинОла,
животные способны учуять даже сквозь метровый слой. Назовите ИХ.

Ответ:
Трюфели.

Зачет:
Черные трюфели.

Комментарий:
Черные трюфели содержат соединение анандамИд, являющееся аналогом
тетрагидроканнабинОла - действующего вещества марихуаны. Свиньи
испытывают болезненное пристрастие к анандамИду, чем и пользуются
сборщики трюфелей. Не уподобляйтесь свиньям!

Источник:
   1. http://www.bbc.com/russian/science/2015/02/150225_vert_ear_truffles_kick
   2. http://en.wikipedia.org/wiki/Truffle_hog

Автор:
Александр Кудрявцев (Николаев)

Вопрос 10:
Знаменитый пейзаж появился благодаря нашествию филлоксЕры, из-за которой
виноградники были выкорчеваны. Этот пейзаж купила ОНА. Назовите ЕЕ.

Ответ:
Компания "Microsoft" [чтецу: мАйкрософт].

Зачет:
Фирма/корпорация "Microsoft"; "Microsoft".

Комментарий:
Речь идет о знаменитой фотографии Чарльза О'Риэра "Безмятежность",
которая используется в качестве стандартных обоев в Windows XP.

Источник:
http://en.wikipedia.org/wiki/Bliss_(image)

Автор:
Александр Кудрявцев (Николаев)

Вопрос 11:
В песне "Спасите наши души" ОНИ названы "рогатой смертью". Первые ОНИ
появились еще в средневековом Китае и представляли собой герметичные
деревянные ящики. Назовите ИХ двумя словами, начинающимися на одну и ту
же букву.

Ответ:
Морские мины.

Комментарий:
Песня Владимира Высоцкого "Спасите наши души" посвящена подводникам
времен Великой Отечественной войны. Старинные китайские морские мины
герметизировали с помощью шпатлёвки.

Источник:
   1. http://vv.kulichki.com/pesni/uxodim-pod-vodu-v.html
   2. http://ru.wikipedia.org/wiki/Морская_мина

Автор:
Юлия Лунёва (Каменское)

Вопрос 12:
Около века назад получили распространение изделия из папье-маше, с
помощью которых удавалось демаскировать врага. Некоторые изделия были
даже снабжены малозаметными гибкими трубочками. Что подавалось через эти
трубочки?

Ответ:
[Сигаретный] дым.

Комментарий:
Чтобы вражеский снайпер демаскировал себя, из папье-маше делали
человеческие головы, которые высовывали из окопов. Для пущей
реалистичности через рот пускали дым, чтобы создалось впечатление, будто
солдат курит.

Источник:
http://en.wikipedia.org/wiki/Sniper#World_War_I

Автор:
Александр Кудрявцев (Николаев)

Вопрос 13:
(pic: 20160743.jpg)
   По мнению одного интернет-пользователя, на розданной вам фотографии
можно увидеть АЛЬФУ. В основном, АЛЬФОЙ занимались общины охотников и
собирателей. Назовите АЛЬФУ двумя словами.

Ответ:
Наскальная живопись.

Комментарий:
На фотографии - известный рестлер и киноактер Дуэйн Джонсон по прозвищу
Скала.

Источник:
http://pikabu.ru/story/mechta_arkheologa_naskalnaya_zhivopis_3165318

Автор:
Ростислав Гимчинский (Черновцы)

Вопрос 14:
Юрия ГрЫмова ОНА со школьных лет вводила в депрессивное состояние. Юрий
видел лишь обессиленную и расстроенную мать, с опустившимися руками и
немым упреком на лице. Назовите ЕЕ.

Ответ:
[Картина] "Опять двойка".

Комментарий:
Грымов пишет, что никогда не был и не хотел быть на месте "главного
героя" этой картины, потому что ему всё время было очень жалко маму.

Источник:
http://www.livejournal.com/media/1718167.html

Автор:
Александр Мудрый (Черновцы)

Вопрос 15:
Эдвард Тайлер находит параллели между кровожадным СкОлем, преследующим
богиню Солнца Соль, и героем известного произведения. Назовите заглавную
героиню этого произведения.

Ответ:
Красная Шапочка.

Комментарий:
Волк Сколь, сын ФЕнрира, преследует Солнце-Соль, которую догонит, когда
настанут Сумерки богов. Красное и круглое Солнце напомнило Тайлеру
головной убор девочки из французской сказки.

Источник:
   1. http://www.universalinternetlibrary.ru/book/23833/ogl.shtml#t23
   2. http://en.wikipedia.org/wiki/Sk%C3%B6ll

Автор:
Игорь Коршовский (Тернополь)

Вопрос 16:
Бейсбольная команда из этого города носит название "Астрос". Она была
создана в 1962 году, но до сих пор испытывает трудности с победой в
Мировой серии. Назовите этот город.

Ответ:
Хьюстон.

Комментарий:
"Хьюстон Астрос" с момента основания до сего дня имеет проблемы с тем,
чтобы победить в финале MLB [эм-эл-би]. Это одна из самых длительных
безвыигрышных серий во всём североамериканском профессиональном спорте.

Источник:
http://en.wikipedia.org/wiki/Houston_Astros

Автор:
Александр Мудрый (Черновцы)

Вопрос 17:
В поездке было запланировано четыре остановки: первая - в Сан-Антонио,
вторая - в Хьюстоне. Ответьте, где была третья остановка, если в
четвертый пункт - город Остин - совершавший поездку человек так и не
попал.

Ответ:
В Далласе.

Зачет:
Даллас.

Комментарий:
Визит в Техас стал последней поездкой президента Джона Кеннеди.

Источник:
https://www.archives.gov/research/jfk/warren-commission-report/chapter-2.html#planning

Автор:
Андрей Остроглазов (Днепр)

Вопрос 18:
Рассуждая о достоинствах и недостатках одной научной теории, принимаемой
ныне большинством ученых, Евгений Кунин вспоминает высказывание
известного англичанина о НЕЙ. Назовите ЕЕ.

Ответ:
Демократия.

Комментарий:
Биолог говорит о теории возникновения жизни на основе молекул РНК:
"РНК-мир - это примерно как демократия по Черчиллю: слабая концепция, с
кучей недостатков, но другие-то гораздо хуже".

Источник:
https://postnauka.ru/books/30297

Автор:
Александр Мудрый (Черновцы)

Вопрос 19:
После того как на чемпионате Европы по футболу исландцы обыграли
англичан, комментатор канала "Украина" пошутил, что в течение недели ОН
произошел дважды. Назовите ЕГО одним словом.

Ответ:
Brexit [чтецу: брЭксит].

Комментарий:
Англия, дескать, еще раз покинула Европу. "Brexit" - сокращение
сочетания слов "Britain" [бритн] - "Британия" и "Exit" [Эксит] -
"выход". 23 июня 2016 года были оглашены результаты референдума о выходе
Великобритании из состава ЕС, а 27 июня футбольная сборная Исландии
сенсационно выбила сборную Англии с Евро-2016.

Источник:
   1. Трансляция матча Англия - Исландия в рамках Евро-2016 на канале
"Украина".
   2. http://ru.wikipedia.org/wiki/Референдум_о_членстве_Великобритании_в_Европейском_союзе
   3. http://ru.uefa.com/uefaeuro/season=2016/matches/round=2000744/match=2018003/index.html

Автор:
Александр Кудрявцев (Николаев)

Вопрос 20:
Когда Ким Чен Иру доложили о либеральных реформах в Китае, тот счел это
предательством социалистических идеалов и назвал китайцев ОТАРОЙ ОВЕЦ,
что из его уст звучало несколько зловеще. Какие два слова мы заменили на
"ОТАРУ ОВЕЦ"?

Ответ:
Свора собак.

Зачет:
Стая собак.

Комментарий:
Гастрономические пристрастия жителей Корейского полуострова широко
известны.

Источник:
John H. Cha, K.J. Sohn. Exit Emperor Kim Jong-il: Notes from His Former
Mentor.
https://books.google.ru/books?id=5m_s-3gIO_UC&pg=PA60#v=onepage&q&f=false

Автор:
Александр Кудрявцев (Николаев)

Вопрос 21:
Словосочетание "ТАКОЙ ИКС" заменяет другие два слова.
   Раскопки свидетельствуют, что ТАКИЕ ИКСЫ нередко использовали в
древнеримских погребальных церемониях. Назовите женщину, для которой в
1934 году был изготовлен ТАКОЙ ИКС.

Ответ:
[Мария] Склодовская-Кюри.

Зачет:
[Мария] Кюри; [Мария] Склодовская.

Комментарий:
ТАКОЙ ИКС - свинцовый гроб. В Древнем Риме использование свинца было
широко распространено во многих сферах. Тело Марии Кюри было сильно
заражено радиоактивными веществами, поэтому ее гроб изнутри защищен
слоем свинца толщиной 2,5 сантиметра.

Источник:
   1. http://www.veronavisita.it/?p=6081
   2. http://www.kp.ru/daily/26424/3297032/

Автор:
Александр Мудрый (Черновцы)

Вопрос 22:
В 1970 году престарелая баронесса заявила, что не позволит визжащим
волосатым обезьянам марать свою фамилию. Напишите эту фамилию.

Ответ:
ЦеппелИн.

Зачет:
Zeppelin.

Комментарий:
В Дании, где жила баронесса Ева фон ЦеппелИн, группе "Led Zeppelin" [лэд
зЭпелин] пришлось выступать под названием "The Nobs" [зэ нобз].

Источник:
http://www.rock-omlet.com/led_zeppelin_4

Автор:
Александр Кудрявцев (Николаев)

Вопрос 23:
В 1969 году Джордж ЛЭзенби сыграл Джеймса Бонда в фильме "На секретной
службе Ее Величества". Актер шутливо сокрушался, что во время съемок
девушки совсем не обращали на него внимание. Ответьте двумя словами, что
в его внешности якобы не нравилось девушкам.

Ответ:
Короткие волосы.

Зачет:
Короткая стрижка.

Комментарий:
Конец 1960-х - это эпоха длинноволосых хиппи и рокеров, а Джеймса Бонда
тяжело представить длинноволосым.

Источник:
   1. Документальный фильм "Всё или ничего: Неизвестная история агента
007".
   2. http://ru.wikipedia.org/wiki/Лэзенби,_Джордж

Автор:
Александр Кудрявцев (Николаев)

Вопрос 24:
На рекламе, предупреждающей о том, что курение убивает, две
переломленные около фильтра сигареты напоминают ЕЕ. Назовите ЕЕ
двухкоренным словом.

Ответ:
Двустволка.

Комментарий:
Сигареты расположены параллельно и напоминают переломленную для
перезарядки двустволку.

Источник:
https://adsoftheworld.com/media/print/shotgun_0

Автор:
Александр Кудрявцев (Николаев)

Вопрос 25:
По мнению Майка Адлера, если средствами логики или математики нельзя
показать, что из суждения выводимы наблюдаемые следствия, то и спорить
нечего. Адлер называет этот принцип "огненным лазерным мечом Ньютона",
который, по его мнению, гораздо опасней... Чего?

Ответ:
Бритвы Оккама.

Комментарий:
Бритва Оккама - принцип, согласно которому из всех объяснений надо
выбирать самое простое, не умножая количества сущностей без надобности.

Источник:
http://www.vshm.science/blog/egorychev/1030/

Автор:
Александр Кудрявцев (Николаев)

Вопрос 26:
В 2014 году Алекс Беллос предложил считать 247 наименьшим скучным
числом, ведь 247 не имело АЛЬФЫ. За три года ситуация изменилась, и
теперь таковым следует считать число 261. Назовите АЛЬФУ.

Ответ:
Статья в [англоязычной] Википедии.

Зачет:
Страница в [англоязычной] Википедии.

Комментарий:
По мнению Алекса Беллоса, у числа 247 не было статьи в Википедии,
поскольку оно ничем не примечательно.

Источник:
http://en.wikipedia.org/wiki/Interesting_number_paradox

Автор:
Александр Кудрявцев (Николаев)

Вопрос 27:
Слова "ТАКАЯ" и "ОНА" являются заменами.
   Мэри РОуч пишет, что дегустатор способен обнаружить в пиве
"больничный" запашок даже в том случае, если ТАКАЯ ОНА использовалась
только для того, чтобы подготовить чаны. Назовите одним словом процесс,
во время которого ОНА становится ТАКОЙ.

Ответ:
Хлорирование.

Комментарий:
ТАКАЯ ОНА - хлорированная вода.

Источник:
М. Роуч. Путешествие еды. http://flibusta.is/b/426312/read

Автор:
Александр Кудрявцев (Николаев)

Вопрос 28:
Отвечая на вопрос, за кого она болеет в матче Роджер ФЕдерер - Стэн
ВАвринка, ТимЕа БачИнски отметила, что быть АЛЬФОЙ - значит быть ТАКОЙ.
Какие слова мы заменили словами "АЛЬФА" и "ТАКАЯ"?

Ответ:
Швейцарка, нейтральная.

Комментарий:
Швейцарская теннисистка не смогла определиться в своих симпатиях в матче
двух земляков. Поэтому сослалась на многовековую швейцарскую
нейтральность.

Источник:
https://twitter.com/TimeaOfficial/status/824579248581120000/

Автор:
Александр Мудрый (Черновцы)

Вопрос 29:
   <раздатка>
   millionaire
   </раздатка>
   Группу "The Color Changin' Click" [зэ кАлэр чЕйнджин клик], что
значит "Кнопка, переключающая цвет кожи", собрал рэпер, в псевдониме
которого мы пропустили буквы. Этот псевдоним образован в результате
слияния слов, означающих "ОН" и "миллионер". Назовите ЕГО.

Ответ:
Хамелеон.

Зачет:
Chameleon.

Комментарий:
Псевдоним рэпера - Chamillionaire [хамиллионер].

Источник:
   1. http://ru.wikipedia.org/wiki/Chamillionaire
   2. http://en.wikipedia.org/wiki/The_Color_Changin'_Click

Автор:
Александр Кудрявцев (Николаев)

Вопрос 30:
В Европу ИХ завезли испанцы, а оттуда, благодаря турецким купцам, ОНИ
распространились по всему континенту. Назовите ИХ.

Ответ:
Индейки.

Зачет:
Индюки; индюшки.

Комментарий:
Именно из-за турецких торговцев, которые ими торговали, в Англии индеек
прозвали "турецкими птицами" ("turkie cocks" [тёрки кокс]), что позже
трансформировалось в современное название "turkey" [тёрки].

Источник:
Дж. Ллойд, Дж. Митчинсон. Книга всеобщих заблуждений.
http://flibusta.is/b/334374/read

Автор:
Александр Мудрый (Черновцы)

Вопрос 31:
Американец Дик Клейн рассматривал варианты "Матадоры" и "Тореадоры", но
в итоге от них отказался, поскольку обладатели многосложных названий
редко занимали высокие позиции. Какой вариант выбрал Клейн?

Ответ:
"[Чикаго] Буллз".

Зачет:
"[Chicago] Bulls"; "Быки".

Комментарий:
Дик Клейн - основатель баскетбольного клуба "Чикаго Буллз". Клейн
называл Чикаго мясной столицей мира из-за огромного количества скотобоен
и подыскивал название, имеющее отношение к крупному рогатому скоту.

Источник:
http://www.nba.com/bulls/news/behind_the_name.html

Автор:
Александр Кудрявцев (Николаев)

Вопрос 32:
В этом вопросе слово "ИКС" заменяет два слова.
   Жившие в Гренландии скандинавы занимались животноводством, но вовсе
не оно было основой их экономики. По образному выражению ПОула ХОльма,
викинги обнаружили в Северной Атлантике ИКС, кишевший моржами и другими
животными. Кому ИКС принадлежал изначально?

Ответ:
Амалфее.

Зачет:
Амалтее; Амальтее.

Комментарий:
Основным промыслом викингов в тех широтах была добыча моржовой кости, и
к XII веку Гренландия стала основным ее поставщиком. А одной из главных
причин упадка гренландских колоний наряду с похолоданием стало падение
цены на моржовую кость в начале XV века.

Источник:
   1. http://polit.ru/article/2016/11/13/ps_greenland/
   2. http://ru.wikipedia.org/wiki/Рог_изобилия

Автор:
Александр Мудрый (Черновцы)

Вопрос 33:
В повести под названием "ИКС" рассказывается о приключениях Сергея
Лисицына, который во время плавания серьезно поссорился с капитаном.
Какие два слова, начинающиеся на одну и ту же букву, мы заменили ИКСОМ?

Ответ:
"Русский Робинзон".

Зачет:
"Российский Робинзон".

Комментарий:
Смутьяна высадили на берегу Охотского моря, оставив ему необходимые для
выживания вещи и припасы. Лисицын не только смог выжить, но даже
разбогател, найдя залежи медной руды и золота в Приамурье.

Источник:
Н. Сибиряков. Русский Робинзон. http://flibusta.is/b/435536/read

Автор:
Александр Мудрый (Черновцы)

Вопрос 34:
(pic: 20160744.jpg)
   Антон Жаворонков проиллюстрировал различные болезни и симптомы,
обыграв их названия. На раздаточном материале мы скрыли бОльшую часть
букв. В каком слове?

Ответ:
Клаустрофобия.

Комментарий:
(pic: 20160745.jpg)
   Буква "я" нарисована в замкнутом пространстве, созданном оставшимися
буквами слова "клаустрофобия".

Источник:
http://pikabu.ru/story/logotipyi_bolezney_4866215

Автор:
Ростислав Гимчинский (Черновцы)

Вопрос 35:
Волейбол - второй по популярности вид спорта в Бразилии. Неофициальное
название волейбольной арены прошлогодней Олимпиады дословно переводится
двумя словами, начинающимися на одни и те же две буквы. Напишите это
название.

Ответ:
"Маленькая МараканА".

Зачет:
"Малая МараканА".

Комментарий:
Спортивный комплекс ЖилбЕрту КардОзу располагается в районе Маракана,
недалеко от знаменитого одноименного стадиона. С этим связано его
неофициальное название - "Мараканазинью".

Источник:
http://ru.wikipedia.org/wiki/Мараканазинью

Автор:
Александр Мудрый (Черновцы)

Вопрос 36:
В июле 1976 года сильный ливень СДЕЛАЛ ЭТО. Когда выяснилось, что
ситуацию исправили с помощью зажигалки, ЭТО СДЕЛАЛИ повторно, после чего
принесли резервную копию. Что такое "СДЕЛАТЬ ЭТО"?

Ответ:
Погасить олимпийский огонь.

Зачет:
Потушить олимпийский огонь.

Комментарий:
Происшествие случилось во время Монреальской олимпиады. Когда ливень
закончился, один из спортивных функционеров зажег олимпийский огонь
своей зажигалкой. Его снова погасили и воспользовались копией огня,
прибывшего в Канаду из Олимпии.

Источник:
http://en.wikipedia.org/wiki/Olympic_flame

Автор:
Александр Кудрявцев (Николаев)

Тур:
6 тур

Дата:
24-Mar-2017

Редактор:
Ярослав Косарев, Сергей Лобачёв (оба - Нижний Новгород), Сергей
Терентьев (Санкт-Петербург)

Инфо:
Редакторы благодарят за тестирование вопросов и ценные замечания: Антона
Волосатова, Екатерину Лобкову, Максима Мерзлякова, Дмитрия Ожигова,
Николая Слюняева, Игоря Тюнькина, Наиля Фарукшина, Татьяну Фёдорову.

Вопрос 1:
Сергей Карякин рассказывает, как в детстве иногда плакал из-за того, что
проигрывал. При этом победы радости ему не доставляли. Какие два слова,
начинающиеся на одну и ту же букву, мы пропустили в этом вопросе?

Ответ:
Самому себе.

Зачет:
Сам себе.

Комментарий:
Карякин играл в шахматы сам с собой, при этом из-за своих зевков в
защите он расстраивался больше, чем радовался успешным атакующим
комбинациям.

Источник:
http://www.mk.ru/social/2016/11/10/seks-pered-partiey-meshaet-semeynye-tayny-shakhmatista-karyakina.html

Автор:
Ярослав Косарев (Нижний Новгород)

Вопрос 2:
По словам ШаоЛань Сюэ, если вы хотите тактично СДЕЛАТЬ ЭТО, вам может
помочь китайский зодиак. Что такое "СДЕЛАТЬ ЭТО"?

Ответ:
Узнать возраст женщины.

Зачет:
Узнать/выяснить возраст женщины/собеседника.

Комментарий:
Выяснив знак китайского зодиака, в большинстве случаев можно почти
наверняка определить год рождения собеседника, поскольку животные
повторяются лишь раз в 12 лет.

Источник:
   1. http://www.ted.com/speakers/shaolan_hsueh
   2. http://www.ted.com/talks/shaolan_the_chinese_zodiac_explained
   3. http://ru.wikipedia.org/wiki/Китайский_зодиак

Автор:
Сергей Лобачёв (Нижний Новгород)

Вопрос 3:
В сериале "Симпсоны" представители одной профессии, обладая большим
количеством свободного времени, стали подрабатывать доставщиками пиццы.
При этом доставка осуществлялась не через дверь. Назовите эту профессию.

Ответ:
Пожарные.

Комментарий:
Пожарные доставляли пиццу клиентам через окно, используя выдвижную
лестницу.

Источник:
Мультсериал "Симпсоны", s25e22.

Автор:
Ярослав Косарев (Нижний Новгород)

Вопрос 4:
Согласно одной теории, человек не может охватить взглядом и моментально
сосчитать более четырех объектов. В качестве подтверждения этой теории
Леонид Крайнов приводит в пример ИХ. В англоязычной мнемонической фразе,
связанной с НИМИ, есть слова "ксилофон", "корова", "молоко". Назовите ИХ
двумя словами.

Ответ:
Римские цифры.

Комментарий:
Первоначально четверка в римской записи изображалась четырьмя
вертикальными черточками, подтверждение чему можно увидеть на множестве
старинных часов. Согласно упомянутой в вопросе теории, отдельный символ
для пятерки упростил восприятие чисел в римской записи. Все слова фразы
"I value xylophones like cows do milk" начинаются с букв, которые
совпадают с обозначениями римских цифр.

Источник:
   1. http://ru.wikipedia.org/wiki/Субитизация
   2. "Знание - сила", 2016, N 8. - С. 110.
   3. http://www.greatmathsgames.com/roman_numerals/roman_numerals_ii.htm

Автор:
Сергей Лобачёв (Нижний Новгород), Сергей Терентьев (Санкт-Петербург)

Вопрос 5:
Филип Хук пишет, что из поколения художников, прославившихся до Первой
мировой войны, мало кто создал что-то значительное после ее окончания.
Далее он мрачно отмечает, что для стоимости картин было бы лучше, чтобы
художники чаще оказывались в НЕМ. Назовите ЕГО словом французского
происхождения.

Ответ:
Авангард.

Комментарий:
Вероятно, автор каламбурит, употребляя это слово, ведь авангард - это
еще и название ряда течений в искусстве, появившихся в начале XX века.
Для стоимости картин некоторых художников, по мнению Хука, было бы
лучше, если бы те умерли на войне. Однако авангарда на многих фронтах
Первой мировой войны не было, так как война была по большей части
окопной.

Источник:
Ф. Хук. Завтрак у Sotheby's. Мир искусства от А до Я.
http://flibusta.is/b/452909/read

Автор:
Сергей Терентьев (Санкт-Петербург)

Вопрос 6:
По забавному совпадению, недалеко от штаб-квартиры компании "Мишлен"
расположен природный объект - парк ИКСОВ. Назовите ИКСЫ.

Ответ:
Вулканы.

Комментарий:
Вулканизация - технологический процесс превращения каучука в резину,
повышающий твердость и эластичность материала. В окрестностях города
находится много потухших вулканов.

Источник:
   1. http://ru.wikipedia.org/wiki/Клермон-Ферран
   2. http://www.vokrugsveta.ru/nauka/article/180916/
   3. http://ru.france.fr/ru/discover/46935

Автор:
Сергей Лобачёв (Нижний Новгород)

Вопрос 7:
Александр Аузан пишет, что в середине XIX века новоприезжие губернаторы
не исполняли своих обязанностей, поскольку почти сразу же отправлялись
ДЕЛАТЬ ЭТО. Что именно делать?

Ответ:
Добывать золото.

Зачет:
Мыть золото; искать месторождения золота; по смыслу с упоминанием
золота/приисков.

Комментарий:
Губернаторы назначались из Вашингтона. И, как и многие другие,
оказавшись в Калифорнии, заболевали "золотой лихорадкой".

Источник:
   1. А.А. Аузан. Экономика всего. Как институты определяют нашу жизнь.
http://flibusta.is/b/346947/read
   2. http://ru.wikipedia.org/wiki/Калифорнийская_золотая_лихорадка

Автор:
Сергей Лобачёв (Нижний Новгород), Сергей Терентьев (Санкт-Петербург)

Вопрос 8:
Кэрри Фишер известна ролью принцессы Леи. На одном рисунке, посвященном
памяти Кэрри Фишер, Бог сворачивает ПЕРВЫЙ во ВТОРОЙ. Назовите ПЕРВЫЙ и
ВТОРОЙ.

Ответ:
Световой меч, нимб.

Зачет:
Лазерный/лучевой/плазменный/джедайский меч, нимб; меч джедая, нимб.

Комментарий:
Свернутый в кольцо световой меч действительно напоминает нимб.

Источник:
   1. https://www.buzzfeed.com/krishrach/11-heartbreaking-cartoons-in-memory-of-carrie-fisher
   2. http://ru.wikipedia.org/wiki/Фишер,_Кэрри

Автор:
Сергей Лобачёв (Нижний Новгород), Сергей Терентьев (Санкт-Петербург)

Вопрос 9:
В исламе считается, что только Аллах может создавать совершенные вещи,
поэтому ИКСЫ ручной работы зачастую содержат едва заметные дефекты.
Назовите ИКС одним словом.

Ответ:
Ковер.

Комментарий:
В большинстве случаев мастера окрашивают один из узелков ковра в
неправильный цвет.

Источник:
   1. http://www.orientalrugexperts.com/deliberate-mistakes-in-handmade-persian-rugs-and-carpets/
   2. http://www.kommersant.ru/doc/303391

Автор:
Сергей Лобачёв (Нижний Новгород), Сергей Терентьев (Санкт-Петербург)

Вопрос 10:
Иван Тянтов пишет, что в конце 30-х - начале 40-х годов в рамках идеи
общественного равенства в репортажах перестали указывать фамилии людей,
СДЕЛАВШИХ ЭТО. Назовите словом английского происхождения того, кому
стремятся СДЕЛАТЬ ЭТО.

Ответ:
Голкипер.

Комментарий:
СДЕЛАТЬ ЭТО - забить гол. Чтобы подчеркнуть успешные результаты
совместной, а не индивидуальной работы, фамилии игроков, забивших гол,
вычеркивались из репортажей. Слова "в рамках" - небольшая подсказка.

Источник:
http://www.sports.ru/tribuna/blogs/field_notes/1063679.html

Автор:
Ярослав Косарев (Нижний Новгород)

Вопрос 11:
Знакомства, заведенные Николаем Гумилёвым в известном месте, Валерий
Шубинский называет ТАКИМИ. ТАКОЙ ИКС на самом деле имеет больше сходств
с полькой и галопом. Назовите ТАКОЙ ИКС.

Ответ:
Собачий вальс.

Комментарий:
Гумилёв завел эти знакомства в кафе "Бродячая собака", где собиралась
тогдашняя петербургская богема.

Источник:
   1. В.И. Шубинский. Зодчий. Жизнь Николая Гумилёва.
http://flibusta.is/b/388894/read
   2. http://ru.wikipedia.org/wiki/Собачий_вальс
   3. http://ru.wikipedia.org/wiki/Бродячая_собака_(кафе)

Автор:
Ярослав Косарев (Нижний Новгород)

Вопрос 12:
Михаил Успенский написал книгу с ожившими идиомами. Так, например, один
народ устал жить без правителя, плюнул на землю и впоследствии создал
себе ИКСА из АЛЬФЫ. Назовите ИКСА и АЛЬФУ.

Ответ:
Князь, грязь.

Комментарий:
Из получившейся грязи народ создал князя.

Источник:
   1. http://old.lgz.ru/archives/html_arch/lg132001/Literature/art12.htm
   2. М.Г. Успенский. Там, где нас нет.
http://books.rusf.ru/unzip/add-2003/xussr_ty/uspenm01.htm?1/49

Автор:
Сергей Терентьев (Санкт-Петербург)

Вопрос 13:
Маргалит Фокс пишет, что если в жизни АЛЬФА была для Артура Эванса
помехой, то в археологической работе помогала. Ироничный Виктор Губарев
связывает прогрессирование АЛЬФЫ с развитием кругозора. Назовите АЛЬФУ.

Ответ:
Близорукость.

Зачет:
Миопия.

Комментарий:
В отличие от большинства людей, Эванс мог на очень близком расстоянии
видеть вещи с почти микроскопической точностью. Согласно афоризму
Виктора Губарева, с расширением кругозора близорукость прогрессирует.

Источник:
   1. М. Фокс. Тайна лабиринта. Как была прочитана забытая письменность.
http://flibusta.is/b/466696/read
   2. http://www.aphorism.ru/comments/7h52ov7s9p.html

Автор:
Ярослав Косарев (Нижний Новгород)

Вопрос 14:
Иронизируя над неумеренностью фон Неймана, его жена говорила, что тот
может сосчитать всё, кроме АЛЬФ. Слово "АЛЬФА" восходит к слову со
значением "тепло". Назовите АЛЬФУ.

Ответ:
Калория.

Комментарий:
Фон Нейман не ограничивал себя в употреблении пищи. Калория -
внесистемная единица количества теплоты, ее название восходит к
латинскому "calor", что переводится как "тепло".

Источник:
   1. У. Айзексон. Инноваторы. Как несколько гениев, хакеров и гиков
совершили цифровую революцию. http://flibusta.is/b/421536/read
   2. http://ru.wikipedia.org/wiki/Калория

Автор:
Сергей Терентьев (Санкт-Петербург)

Вопрос 15:
Герой фильма, действие которого происходит в первой половине XIX века в
США, убил человека. Для того чтобы отвести от себя подозрения, он
проделал с трупом некую операцию. Ответьте двумя словами, начинающимися
на одну и ту же букву: что именно он сделал?

Ответ:
Снял скальп.

Зачет:
Срезал скальп.

Комментарий:
Речь идет о фильме "Выживший". Герой хотел, чтобы все подумали, что это
убийство было делом рук индейцев.

Источник:
Х/ф "Выживший" (2015), реж. Алехандро Гонсалес Иньярриту.

Автор:
Ярослав Косарев (Нижний Новгород)

Вопрос 16:
Согласно журналу "Discovery" [дискАвери], врЕменные меры, направленные
на повышение рождаемости, помогли ИМ заселить весьма суровый край.
Назовите ИХ.

Ответ:
Мормоны.

Комментарий:
Первые поселения в Юте мормоны основали в 1847 году. А уже в 1890 году
мормонская церковь отказалась от многоженства.

Источник:
"Discovery", 2016, N 11. - С. 73.

Автор:
Сергей Лобачёв (Нижний Новгород)

Вопрос 17:
В романе, действие которого происходит в конце XIX века, известный
человек жалуется на свою судьбу. Свое несчастье он связывает с тем, что
во время беременности его мать напугал ИКС. В одной из сцен первой книги
серии Гарри Поттер был частично ограничен в движениях, поскольку был
ИКСОМ. Какое слово мы заменили на ИКС?

Ответ:
Слон.

Комментарий:
Героем романа является Джозеф Меррик, который за уродливое тело получил
прозвище "Человек-слон". Гарри Поттер был шахматным слоном.

Источник:
   1. Ф. Пальма. Карта времени. http://flibusta.is/b/265941/read
   2. Дж. Роулинг. Гарри Поттер и философский камень.
http://flibusta.is/b/469357/read

Автор:
Ярослав Косарев (Нижний Новгород)

Вопрос 18:
Фрэнсис Эшкрофт пишет, что в начале XIX века многие считали
электричество формой огня, чем, вероятно, и объясняется название
известного произведения. Какого?

Ответ:
"Франкенштейн, или Современный Прометей".

Зачет:
"Франкенштейн".

Комментарий:
Электричество, в том числе и молнии, обыватели считали формой огня. Хотя
в романе и не говорится, что именно молния оживила монстра, есть все
основания полагать, что ученый использовал электричество. В предисловии
Мэри Шелли красочно описывает гальванизацию, а в самом начале романа
Виктор впечатляется молнией, которая разнесла в щепки дуб, и узнаёт у
своего отца об электричестве.

Источник:
Ф. Эшкрофт. Искра жизни. Электричество в теле человека.
https://books.google.ru/books?id=DmZoBgAAQBAJ&pg=PA27#v=onepage&q&f=false

Автор:
Сергей Лобачёв (Нижний Новгород)

Вопрос 19:
Точка Немо - наиболее удаленное от суши место в океане. Эта особенность
активно используется некоторыми странами, и окрестности точки Немо
получили прозвище "ОНО". Назовите ЕГО тремя словами, начинающимися на
одну и ту же букву.

Ответ:
Кладбище космических кораблей.

Комментарий:
Большинство космических держав стараются утилизировать свои
искусственные спутники в безлюдном районе точки Немо.

Источник:
   1. http://www.bbc.com/earth/story/20161004-the-place-furthest-from-land-is-known-as-point-nemo
   2. http://www.popsci.com/this-is-where-international-space-station-will-go-to-die
   3. http://www.amusingplanet.com/2017/02/point-nemo-spacecraft-cemetery.html

Автор:
Сергей Лобачёв (Нижний Новгород)

Вопрос 20:
Первые представители профессии старались защитить свои руки от
переохлаждения и поэтому надевали перчатки. Место, где хранились эти
перчатки, англичане называют "glove box" [глав бокс]. А как это место
называем мы?

Ответ:
Бардачок.

Комментарий:
Первые автомобили имели открытый верх, и встречный поток воздуха мог
обморозить руки.

Источник:
http://en.wikipedia.org/wiki/Glove_compartment

Автор:
Сергей Лобачёв (Нижний Новгород)

Вопрос 21:
Успеху персонажа одного произведения позавидовали друзья, и его трофей
постепенно "съежился". Хотя обычно в НИХ трофей только растет. Назовите
ИХ двумя словами, начинающимися на одну и ту же букву.

Ответ:
Рыбацкие рассказы.

Зачет:
Рассказы рыбаков; рыбацкие речи; речи рыбаков.

Комментарий:
Мальчик поймал большую рыбу, но друзья позавидовали ему, и вскоре
благодаря сарафанному радио улов съежился до размеров жалкого пескарика.

Источник:
Дж. Оруэлл. Глотнуть воздуха. http://flibusta.is/b/452071/read

Автор:
Сергей Терентьев (Санкт-Петербург)

Вопрос 22:
Слова "ИКС" и "ИГРЕК" в этом вопросе являются заменами.
   В Средневековье люди считали, что животные имеют двойников в других
сферах. К примеру, лошадь - животное земное, а потому существование ИКСА
позволяло верить в реальность ИГРЕКА. Назовите ИГРЕКА.

Ответ:
Пегас.

Зачет:
Крылатый конь; гиппогриф.

Комментарий:
Каждое животное представляло свою стихию: лошадь - земную, морской конек
- водную, а Пегас - небесную.

Источник:
Т. Уайт. Средневековый бестиарий. Что думали наши предки об окружающем
их мире. http://flibusta.is/b/356817/read

Автор:
Сергей Лобачёв (Нижний Новгород)

Вопрос 23:
В 1626 году после смерти матери сёгуна власть в женской половине замка
перешла к Касуга Но Цубоне, которая была АЛЬФОЙ. Чтобы такое не
повторилось в дальнейшем, во время работы АЛЬФЫ были обязаны носить
маску. Какое слово мы заменили на АЛЬФУ?

Ответ:
Кормилица.

Зачет:
Мамка; заботница; млекопитательница; кормилка; сухотница;
поилица-кормилица; благодетельница.

Комментарий:
Сёгун Токугава Иэмицу рос под опекой своей кормилицы, и его
привязанность позволила той впоследствии занять высокое положение. Чтобы
незнатные женщины не могли повторить ее путь, кормилиц заставили носить
маску.

Источник:
А.Ф. Прасол. От Эдо до Токио и обратно. Культура, быт и нравы Японии
эпохи Токугава. http://flibusta.is/b/312080/read

Автор:
Ярослав Косарев (Нижний Новгород)

Вопрос 24:
Эпидемиологи выяснили, что АЛЬФЫ снижают рост заболеваемости и скорость
распространения инфекций. В США первая крупная АЛЬФА произошла в 1877
году. Какие два слова, начинающиеся на соседние буквы алфавита, мы
заменили словом "АЛЬФА"?

Ответ:
Забастовка железнодорожников.

Комментарий:
Население становится менее мобильным, и скорость распространения
эпидемии замедляется.

Источник:
   1. "Наука и жизнь", 2016, N 8. - С. 24.
   2. Г.Н. Севостьянов. История США.
https://books.google.ru/books?id=1cb-AgAAQBAJ&pg=PA482#v=onepage&q&f=false
   3. http://en.wikipedia.org/wiki/Great_Railroad_Strike_of_1877

Автор:
Сергей Лобачёв (Нижний Новгород)

Вопрос 25:
По словам Ги де Мопассана, в XIX веке ревнивцы иногда по утрам
завязывали на НЕМ сложные бантики. Назовите ЕГО одним словом.

Ответ:
Корсет.

Комментарий:
Вечером по бантикам можно было проверить, снимался ли корсет.

Источник:
   1. http://www.epatage-club.ru/moda-i-stil/bantiki-na-korsjetje.html
   2. А.А. Васильев. Судьбы моды. http://flibusta.is/b/353498/read

Автор:
Ярослав Косарев (Нижний Новгород)

Вопрос 26:
Группа профессора Томаса Вичека проанализировала большое количество
видеозаписей и пришла к выводу, что для ЕЕ запуска достаточно двадцати
пяти человек. Назовите ЕЕ.

Ответ:
Волна.

Зачет:
Футбольная волна; волна болельщиков; волна фанатов; волна на стадионе;
мексиканская волна; волна на трибунах.

Комментарий:
Для того чтобы волна "пошла", необходимо набрать критическую массу
скоординированно действующих болельщиков.

Источник:
https://www.kinnet.ru/cterra/463/20519_3.html

Автор:
Сергей Лобачёв (Нижний Новгород)

Вопрос 27:
При встрече с Нэнси Уэйк первыми словами командира партизанского отряда
было "Ах, если бы все деревья приносили столь великолепные плоды!".
Причиной этого послужил не самый удачный ИКС Нэнси. Назовите ИКС тремя
словами.

Ответ:
Прыжок с парашютом.

Комментарий:
Нэнси Уэйк зацепилась стропами парашюта за дерево. Командир отряда,
обнаруживший ее, галантно пошутил, однако Нэнси не любила, когда мужчины
разговаривали с ней в подобном тоне, и ответила грубо: "Только давайте
без вашей французской хрени!".

Источник:
Б. Акунин. Нечеховская интеллигенция. Короткие истории о всяком разном.
http://flibusta.is/b/468587/read

Автор:
Сергей Лобачёв (Нижний Новгород)

Вопрос 28:
Съехав от жены, Гомер Симпсон позвонил домой и, услышав в записи
автоответчика ЕЕ, понял, что дело обстоит серьезно. Назовите ЕЕ.

Ответ:
Девичья фамилия Мардж.

Зачет:
Девичья фамилия жены; девичья фамилия; фамилия Бувье; Бувье.

Комментарий:
Дело пахнет разводом, раз Мардж сменила на автоответчике фамилию Симпсон
на девичью - Бувье.

Источник:
Мультсериал "Симпсоны", s27e01.

Автор:
Ярослав Косарев (Нижний Новгород)

Вопрос 29:
Этот человек ложился на рельсы перед поездами, которые везли
радиоактивные отходы к месту утилизации. В 1997 году его похоронили на
земле коммуны Сент-Андре-де-Кюбзак в нарушение его воли. Назовите этого
человека.

Ответ:
[Жак-Ив] Кусто.

Комментарий:
Кусто протестовал против захоронения отходов в океане. Он завещал
похоронить себя в океане, но жена проигнорировала эту просьбу.

Источник:
   1. https://www.otpusk.com/articles/794/
   2. "Gala. Биография", 2016, N 12.
   3. http://ru.wikipedia.org/wiki/Сент-Андре-де-Кюбзак

Автор:
Ярослав Косарев (Нижний Новгород)

Вопрос 30:
Катастрофические ситуации на шестой и девятой лунках на сленге
гольфистов получили два прозвища, совпадающие с именами собственными.
Назовите любое из этих прозвищ.

Ответ:
Хиросима, Нагасаки.

Комментарий:
Бомбардировки этих городов произошли шестого и девятого августа
соответственно.

Источник:
http://www.golftoday.co.uk/19th/humour/shot_names.html

Автор:
Сергей Лобачёв (Нижний Новгород)

Вопрос 31:
В XVII веке по всей Японии сажали деревья эноки, которые можно считать
аналогом ИКСОВ. В Российской империи про ИКС ходила загадка: "Нем и
глух, а счет знает". Назовите ИКСЫ двумя словами.

Ответ:
Верстовые столбы.

Зачет:
Дистанционные столбы; почтовые столбы.

Комментарий:
Эноки сажали на определенном расстоянии друг от друга, и путник мог
отдохнуть в тени дерева.

Источник:
   1. А.Ф. Прасол. От Эдо до Токио и обратно. Культура, быт и нравы
Японии эпохи Токугава. http://flibusta.is/b/312080/read
   2. http://www.zaist.ru/news/vsyakaya_vsyachina/verstovye_stolby/

Автор:
Ярослав Косарев (Нижний Новгород), Сергей Терентьев (Санкт-Петербург)

Вопрос 32:
В колониальной Англии за чай расплачивались шестипенсовыми монетами,
которые при этом играли роль... Чего именно?

Ответ:
Гирек весов.

Зачет:
Гирек для весов; гирек; грузиков на весах; разновеса.

Комментарий:
В колониальную эпоху за порцию чая платили шестипенсовой монетой - их
уравнивали на весах. Шесть пенсов весили 2,8 грамма. Но для простоты со
временем это количество округлили до трех граммов.

Источник:
http://www.vokrugsveta.ru/article/241596/

Автор:
Ярослав Косарев (Нижний Новгород), Сергей Терентьев (Санкт-Петербург)

Вопрос 33:
В Норвегии в основном используются возобновляемые источники энергии.
Сигурд Квикне сетует, что в Норвегии выросло целое поколение, привыкшее
не ДЕЛАТЬ ЭТО. Как называется ежегодное событие, во время которого
принято ДЕЛАТЬ ЭТО?

Ответ:
Час Земли.

Комментарий:
ДЕЛАТЬ ЭТО - выключать свет. Так как электричество относительно дешевое,
то свет особо не экономят.

Источник:
http://www.vokrugsveta.ru/article/239131/

Автор:
Ярослав Косарев (Нижний Новгород)

Вопрос 34:
По одной из версий, появление известного выражения связано с тем, что до
XVIII века люди всегда носили АЛЬФУ с собой, а потому ТАКАЯ АЛЬФА
являлась показателем статуса владельца. Какие два слова мы заменили
словами "ТАКАЯ АЛЬФА"?

Ответ:
Серебряная ложка.

Зачет:
Серебряная ложечка.

Комментарий:
Традиция сервировать стол набрала в Европе популярность лишь к началу
XVIII века. Поэтому люди того времени носили ложки при себе. Серебряная
ложка являлась статусным атрибутом, что привело к появлению выражения
"родиться с серебряной ложкой во рту", означающего "родиться в богатой,
обеспеченной семье".

Источник:
http://en.wikipedia.org/wiki/Silver_spoon

Автор:
Сергей Лобачёв (Нижний Новгород)

Вопрос 35:
Аббат монастыря святого Фомы поощрял ЕГО занятия, надеясь благодаря
увеличению поголовья овец и урожая фруктов вылезти из долгов. Назовите
ЕГО.

Ответ:
[Грегор] Мендель.

Комментарий:
Грегор Мендель является основоположником теории наследственности,
которую он вывел, разводя горох в монастырском саду.

Источник:
   1. С. Кин. Синдром Паганини и другие правдивые истории о
гениальности, записанные в нашем генетическом коде.
http://flibusta.is/b/426306/read
   2. http://ru.wikipedia.org/wiki/Мендель,_Грегор_Иоганн

Автор:
Сергей Терентьев (Санкт-Петербург)

Вопрос 36:
Дэниел Ергин пишет, что в конце XIX века в один из городов Российской
империи стекалось множество людей, мечтающих о собственном фонтане.
Назовите город, о котором идет речь.

Ответ:
Баку.

Комментарий:
Речь идет о нефтяных фонтанах. Многие люди покупали земельные участки в
надежде обнаружить на них нефть.

Источник:
http://www.dobi.oglib.ru/bgl/332/1716.html

Автор:
Ярослав Косарев (Нижний Новгород)


FreeBSD-CVSweb <freebsd-cvsweb@FreeBSD.org>